terça-feira, 6 de novembro de 2018

Você tem que parar de se prender em dogmas! Não é só porquê você admira Einstein ou fez seus estudos fundamentados nas teorias dele. Ele não foi mais gênio que o Maxwell, tampouco Newton

Não são dogmas. São opiniões. A minha é de que Einstein foi mais genial do que Newton e Maxwell. É o que eu acho. Já estudei as contribuições dos três para a Física. Não só estudei como lecionei. Portanto conheço bem.

no caso do universo ser eterno, que o tempo seria uma reta, então não há big bang pois big bang marca o início da passagem do tempo. mas mesmo assim, a teoria do "big bang" é válida, apenas desconsiderando o início da passagem do tempo, pois não teve início nenhum, se for eterno. Certo?

Errado. A consideração da existência do Big Bang é associada à de que o tempo teve um início. Não há Big Bang em um tempo pré existente.

existe uma definição para "reta" em geometria?

Não. Reta é um conceito primitivo, do mesmo modo que ponto e plano. O que se tem é um conceito e não uma definição. O conceito de reta é o de uma linha infinita nos dois sentidos, que permaneça sempre com a mesma direção. Mas, aí, vem a questão de saber o que seja direção, que também é um conceito primitivo.

"E ainda se vier noites traiçoeiras, se a cruz pesada for Deus estara contigo,o mundo pode ate querer fazer vc chorar mas Deus te quer sorrindo"

Como você pode saber o que o deus, caso exista, queira? Já vi muita gente pondo na boca do deus seus pensamentos, sem nenhuma comprovação de que o deus que supõem existir, de fato, o tenha dito. Portanto não considero que, se existir algum deus, ele diga nada para ninguém.

mas no caso do tempo ser uma "reta", o momento zero que vem depois do infinito negativo e antes do positivo pode ser escolhido arbitrariamente como o big bang também?

Se o tempo for uma reta e não uma semi-reta, sua origem pode se colocar em qualquer instante. Todavia os dados observacionais indicam que o tempo não seja uma reta e sim uma semi-reta, ou seja, há uma origem dos tempos em relação à qual não existem momentos anteriores. No caso do tempo ser uma reta, não haveria big bang.

Mas, Ernesto, o Hamiltoniano é definido usando uma Lagrangeana. Sendo assim, seria possível construir uma MQ somente em termos da Lagrangeana, não?

Nunca vi isso. Provavelmente isso já foi aventado por alguém e verificado que não funciona, tanto que não se estabeleceu, como foi o caso da quantização da mecânica hamiltoniana.
https://en.wikipedia.org/wiki/Canonical_quantization
Todavia, em Teoria Quântica de Campos é usual se começar com uma densidade lagrangiana clássica e se fazer sua quantização.
https://en.wikipedia.org/wiki/Lagrangian_(field_theory)

Você considera que Maxwell foi um gênio?

Não acho que tenha sido um gênio. Gênio é uma raridade muito grande. Na Física eu só considero dois: Newton e Einstein. Certamente que Maxwell e outros físicos notáveis foram pessoas extremamente inteligentes e dedicadas ao estudo, com o que conseguiram fazer inúmeras descobertas.

o universo pode ter um fim mesmo sendo eterno. mas se é eterno, como chegamos no presente? a cosmologia ainda não tem essa resposta e essa questão é filosófica.

Ser eterno para o passado é completamente independente de ser eterno para o futuro. Pode ser um sem o outro, qualquer deles. Ou ambos. Ou nenhum deles. Não há problema nenhum em ser eterno para o passado e haver o agora. O que não pode é considerar que o Universo tenha surgido em algum momento infinitamente afastado para o passado. Se assim o fosse, não teria havido tempo para se chegar agora. Mas o Universo pode, perfeitamente, não ter surgido nunca, ou seja, para qualquer momento que se considere há um momento anterior, sem ter havido um primeiro. Então a linha do tempo não é uma semi-reta e sim uma reta, que vai de menos infinito para mais infinito, ficando o momento zero como o momento presente. Todavia, os dados observacionais apontam no sentido de que o Universo tenha tido um momento inicial, mesmo que isso não seja uma necessidade. Esse momento inicial teria se dado há treze bilhões e oitocentos e vinte milhões de anos.

E o Estado não tem o direito (e nem devia) de fazer o que quiser com a propriedade privada dos outros, sendo que não há coação. Alguém é obrigado a entrar numa boate? Não. Agora, o imposto é uma coação. Eu não concordo em pagar imposto; não assinei nenhum contrato! Meu direito é respeitado?

Os impostos são estabelecidos por lei que é votada pelos deputados eleitos pelo povo para os representar. Não é preciso que cada um, individualmente, concorde em pagar impostos para ter que pagar. A lei estabelece isso é tal fato é perfeitamente legítimo. Isso é o "Contrato Social" que a população faz com seu governo, para que ele cuide dos interesses gerais e não dos particulares. Até que a humanidade seja tão perfeitamente civilizada que dispense ser governada, porque todos fazem o que precisa ser feito sem coação, os governos ainda serão necessários. E, como ainda existe dinheiro, os governos precisam dele para fazer o que devem. Portanto o imposto não é roubo não. Roubo é o que muitos políticos fazem com o dinheiro arrecadado. Mas isso é conjuntural e não estrutural. O fato de políticos corruptos tomarem o dinheiro do governo para si não invalida o fato do governo precisar de dinheiro para governar.

Ernesto, você se intitula físico mesmo sem ter doutorado e publicar artigos em periódicos com frequência? Ou matemático tendo feito apenas licenciatura?

Sim, porque era assim que se considerava quando me formei, em 1971. Atualmente já não é mais assim. As Universidades contratavam licenciados porque não havia mestres e doutores para atender as necessidades.

Ora, se é privado não é público! A questão não é se é justo ou injusto. Se há um grupo de homens que não concorda em pagar um ingresso mais caro, logo iria surgir uma boate que esse grupo I. pague o mesmo ou II. pague menos. O próprio mercado resolveria esse problema. O Estado tem que ficar de fora!

Não! O estado tem que intervir sempre que a iniciativa privada descumprir a lei ou a constituição. É seu dever em nome dos cidadãos. A iniciativa privada não tem o direito de fazer tudo o que quiser.

Acredita que o Parlamentarismo seria um dos caminhos para resolver os problemas do Brasil?

Sim, sou totalmente a favor. No parlamentarismo governos podem ser dissolvidos, novas eleições podem ser convocadas, tudo isso dentro da normalidade democrática.

Uma coisa é o Estado roubar homens e mulheres via imposto e favorecer as mulheres (como faz com a campanha do câncer de mama, que tem mais investimento que a do câncer de próstata; delegacia para mulher etc...). Agora, o empresário obrigou alguém a entrar na sua boate? É uma propriedade privada!

Mesmo o que seja privado tem que estar de acordo com a lei. Um restaurante, um bar, um teatro, um cinema, um salão de baile, uma igreja, um bordel, mesmo sendo privados são locais públicos e não se pode discriminar ninguém. A única exceção é em razão de idade, para a qual se pode estabelecer um valor mínimo para acesso.

Gosta de ouvir música conforme o seu estado de espírito,ou seja,mais alegre quando está e mais feliz? Ou usa a música para mudar o seu estado de espírito,ouvindo música alegre quando está triste?

Em geral eu prefiro ouvir a música que esteja consoante meu estado de espírito. Mas, às vezes, ouço música triste estando alegre e música alegre estando triste.

https://goo.gl/C3bxKt Pronto! O Estado vai novamente se intrometer nos negócios dos outros. Sou totalmente a favor do empresário contratar só homens ou mulheres, se quiser; de colocar preço de ingresso diferente para homens e mulheres. Não é projeto social; é negócio! Não gostou, não vá, caralho!

Eu concordo com essa decisão judicial. Os negócios não podem ferir a Constituição que estabelece direitos iguais para todos, independentemente do gênero. Sua concepção é preconceituosa. Não se pode impedir ou dificultar o acesso de ninguém a nada pelo gênero, a raça, a religião, o nível social ou o que for. Por exemplo, não é constitucional que prédios tenham elevadores diferentes para servidores e moradores e suas visitas. O estado tem a obrigação de, por suas leis, promover a igualdade.

Baseado na sua experiência como professor qual a melhor metodologia para lecionar matemática e física nos níveis fundamentais , médio e superior?

Em todos esses níveis a melhor metodologia é a de provocar a aquisição de conhecimento pelo próprio estudante, que vai descobrir os fatos dessas ciências em atividades planejadas e conduzidas pelo professor, que não expõe os resultados prontos, mas leva os estudantes a concluírem por si mesmos. Especialmente na Física, mas mesmo na Matemática, é preciso fazer uso de experimentos feitos pelos próprios estudantes para que vejam o que acontece e tirem suas conclusões. Para tal é muito importante que eles treinem a induzir leis e a demonstrar teoremas por si mesmo e que resolvam muitos exercícios e problemas com dados literais e não numéricos. Mesmo os que possuam dados numéricos devem ser resolvidos de forma literal antes de se substituir os valores numéricos na expressão final que responde a questão. Em suma, é preciso fazer a turma "fundir a cuca".

Professor, ver diferença entre o que é objetivo e o que é intersubjetivo?

Objetivo é algo que não dependa da opinião das pessoas, enquanto subjetivo é o que dependa. Intersubjetivo é algo que dependa da opinião das pessoas mas ela seja coincidente entre várias pessoas. Em muios casos se pode tomar a intersubjetividade como se fosse uma objetividade, por não se ter uma verificação objetiva da validade do que seja dito.

As pessoas que discordam do aquecimento global estão equivocadas ? Visto que a própria corrente negacionista apoia essa tese, contudo sem que a intervenção antrópica esteja diretamente relacionada. Também, que outra explicação há para o derretimento de tantas cordilheiras?

Que o aquecimento global está acontecendo é uma constatação que não se pode contestar. O que se contesta é se ele é devido principalmente à atividade do homem ou a causas naturais. Isso é controverso e há posições em ambos os lados, com boa argumentação. Por outro lado, o aquecimento global ora observado é uma subida de dente de uma serra que está inclinada para baixo e levará, em poucos milênios, a uma nova era glacial. A clima da Terra varia em vários ciclos dentro de ciclos, o de milhões de anos, milhares de anos, dezenas de anos.

Professor, acredita que todos somos inteligentes porem em campos da inteligência diferentes? Ou seria muito disney pensar isso?

Não somos não. Há quem seja burro mesmo, em todos os campos de inteligência. Como há quem seja muito inteligente em todos os campos também. E há quem seja mais inteligente em alguns campos e menos em outros. Este é o caso da maioria. Todavia o QI médio da humanidade é o QI 100, pela própria construção do teste, que é feito para média 100 e desvio padrão 15. Isto significa que metade da humanidade teria QI igual ou menor que 100 e metade maior. Mas um QI 100 não é nada inteligente. Para se dizer que alguém seja inteligente, para mim, teria que ter, pelo menos, QI 110.
https://pt.wikipedia.org/wiki/Quociente_de_intelig%C3%AAncia

Você já foi filiado ou simpatizante de alguma partido político? Você acha que a política é importante?

Nunca me filiei a partido nenhum, pois sou muito independente em minhas ideias políticas e econômicas e nenhum deles corresponde a meu pensamento. Todavia já simpatizei com o PT e me desapontei completamente. Outro que também simpatizei foi o PDS (antigo PTB, do Brizola), mas hoje não sei não.

por mais que se procure diversas definições de diversas palavras num dicionário, todas elas serão redutíveis a uma só? definições são arbitrárias e irrefutáveis, mas há diferentes modos de expô-las?

A questão é que, na linguagem comum, nem sempre existe uma definição consensual para cada palavra, de modo que uns e outros consideram que ela tenha significados diferentes. Em ciência, contudo, cada termo é definido de modo preciso e sem nenhuma ambiguidade. No entanto, há autores de livros didáticos que não exaram a definição precisa de alguns termos, talvez porque consideram que seja complicada, o que, de fato, costuma acontecer. Só que não se pode ensinar algo errado porque seja complicado. Tem que ensinar do modo complicado mesmo. Além da definição precisa, é necessário que sejam feitos comentários abrangentes sobre o conceito do termo que esteja sendo definido, para que não fique nenhuma dúvida a respeito. Do mesmo modo que, quando se apresenta uma lei científica ou um teorema, tem-se que explicar bem direitinho os casos em que ela se aplica e aqueles em que não se aplica, em todos os pormenores. Isso, aparentemente, também complica, mas não se pode furtar a considerar tudo, sob pena do aprendizado acontecer de forma imperfeita.

essa é difícil. a matemática lida com verdades absolutas? isso a torna irrefutável?

Se por verdade absoluta se estiver entendendo a adequação de algo à realidade do mundo, nem sempre a matemática trabalha assim. Há muitas teorias matemáticas totalmente abstratas e não correspondentes à realidade do mundo que, nem por isso, deixam de ser matematicamente válidas. Como a Geometria de Lobatchevsky, por exemplo. A Matemática se fundamenta em definições, que são arbitrárias e deduz teoremas baseados nas definições e em axiomas, que também são arbitrários. Todavia a utilidade prática da Matemática (que não é uma necessidade para ser válida) vai acontecer quando as definições e axiomas forem conformes o que, de fato, acontece no mundo. A Matemática é refutável sim. Por exemplo, a Geometria Euclideana, que é uma teoria matemática totalmente consistente e válida, não corresponde à geometria do Universo em várias situações, como no entorno de uma estrela, como o Sol, em que o espaço não é euclideano.

Professor, o senhor acha que vocação existe? Fantasiam muito isso na hora de prestar o vestibular, mas vejo muita gente realizada profissionalmente, estas que em sua maioria seguem a profissão dos pais, é muita coincidência, não? Somos o que pensamos ser? Ou pensamos que somos o que realmente somos?

Existe sim. Não especificamente para uma profissão, mas pessoas diferentes possuem pendores para atuar em áreas diferentes, como indústria, comércio, esportes, política, finanças, religião, magistério, engenharia, medicina, direito, teatro, música, artes plásticas, literatura, pesquisa em ciências exatas, biológicas ou humanas, informática ou outras atividades. Todavia uma pessoa pode se dedicar a algo para o que não tenha tendência natural e, mesmo assim, se dar bem, especialmente no caso de ser uma atividade que sua família já se dedica. O melhor, contudo, é atuar naquilo em que se sente bem, que dá prazer, em que se realiza.

Quero conhecer a Relatividade e a Mecânica Quântica formalmente, porém, estou um pouco perdido nessa jornada. Poderia, então, recomendar uma bibliografia partindo da física básica até as teorias que citei (Com as devidas citações da matemática necessária)?

Para a Física Básica eu recomendo, no nível do Ensino Médio, a coleção do PSSC (Physical Science Study Committee), em quatro volumes, editado em português pela Edart, bem como o seu volume de Tópicos Avançados (http://www.ebah.com.br/user/AAAABPKIgAL/ernesto-von-ruckert ).
No nível básico do Ensino Superior eu recomendo o Curso de Física de Berkeley, cujos primeiros dois volumes foram traduzidos para o português pela Edgard Blücher. Os outros três existem em espanhol, pela Reverté e, inclusive, envolvem uma introdução de Física Quântica. Outro bom, também, é o do Alonso & Finn, que agora tem uma edição em volume único em português, de Portugal.
Especificamente na Física Moderna (Quântica, Atômica, Molecular, Nuclear e Relatividade Especial) eu recomendo os livros do Eisberg - Fundamentos de Física Moderna, Beiser - Conceitos de Física Moderna, Acosta, Cowan & Grahan - Curso de Física Moderna e Eisberg & Resnick - Física Quântica.
Infelizmente não existem livros introdutórios de Relatividade Geral. O mais acessível é o do Ohanian - Gravitation and Spacetime.
A Matemática necessária é só o Cálculo Diferencial e Integral, a Álgebra Linear e as Variáveis Complexas. Contudo é interessante dar uma estudada em um livro de Métodos Matemáticos da Física, como o do Arfken & Weber.
View more

em topologia, um triângulo é redutível a um circulo, se deformar esse triângulo, mas para ser redutível, então a área desse triângulo deve ser igual a do círculo?

Não. Topologia não cuida de distâncias, áreas, volumes e nenhuma medida em relação às figuras. Apenas em posições e regiões.

Não entendo porque, no universo de conhecimento que possui, se concentra em poucos assuntos. Como você mesmo diz, se pretende espancar a ignorância, seria muita mais viável diversificar os temas abordados.

Mas eu não me concentro em poucos assuntos. Respondo sobre Física, Matemática, Cosmologia, Astronomia, Astrofísica, Engenharia, Filosofia (Metafísica, Epistemologia, Lógica, Ética, Estética, Filosofia da Mente), Religião, Sociologia, Neurociências, Evolução, Informática, Filosofia Política, Filosofia Econômica, Educação, Música, Pintura, Literatura, Comportamento, Relacionamentos e outros assuntos. Depende do que a pessoa me perguntar e eu souber responder. O que não tenho conhecimento é Direito, Administração, Finanças, Agricultura, Medicina, Esportes e vários outros assuntos que não entendo.

a força gravitacional pode ser expressa assim: =-mMGr'/r³, sendo r' um vetor. Mas ela é redutível a mMG/r²?

Sim, uma vez que, sendo r' o vetor posição ele vale r.ř, em que ř é o unitário adimensional na direção e sentido do vetor r'. Assim, na expressão de força gravitacional se tem F = - GMmrř/r³ = - GMmř/r².

por que a equivalência de massa inercial e gravitacional é válida apenas localmente. o que quer dizer "localmente"?

A equivalência local não é entre massa inercial e gravitacional. Essa é geral, tanto local quanto globalmente. O que é local é a equivalência entre um campo gravitacional e um referencial acelerado, que não é global porque um campo gravitacional nunca é uniforme, possuindo gradiente que aponta no sentido do campo, enquanto a aceleração de um referencial não inercial ou é uniforme ou tem um gradiente que aponta em sentido oposto a ela mesma. Localmente se refere a algo que seja válido em cada ponto mas não em uma região extensa.

velocidade e inércia não são propriedades do movimento?

Velocidade é uma propriedade do movimento, como aceleração. Inércia é uma propriedade da matéria. Velocidade e aceleração também são grandezas. A grandeza correspondente à inércia, que mede o seu valor, é chamada de massa inercial.

Você gostaria de morar em Cuba?

De modo nenhum. Não concordo nem um pouco com o regime político e econômico de Cuba. É tão ruim quanto o capitalismo. E lá não existe democracia, que considero o único sistema político legítimo, a haver estado e governo. Lá é uma ditadura. E o socialismo de estado é perverso. Não tem nada de comunismo.

Por que o conceito de inércia mesmo sendo simples e intuitivo é considerado uma Lei da física?

O conceito de inércia não é uma lei da Física. A "Lei da Inércia", não descreve o conceito de inércia, que tem que ser estabelecido separadamente. O que a lei diz é que, existem referenciais nos quais um corpo sujeito a uma força resultante nula não possuirá aceleração. Tais referenciais são ditos inerciais. O conceito de inércia é o de uma propriedade da matéria pela qual um corpo só modificará a sua velocidade vetorial, medida em um referencial inercial, se estiver sujeito a uma força resultante não nula.

a força normal é conservativa?

A força normal é a componente da resultante perpendicular à velocidade. Portanto ela não é, necessariamente, uma força de interação e sim o resultado da composição de várias forças, dentre as quais podem haver conservativas e não conservativas. Note que força resultante não é uma força de interação e sim uma força fictícia que, se existisse sozinha, provocaria o mesmo efeito no movimento do que o sistema de todas as forças que atuem sobre um corpo. No caso, por exemplo, de um satélite em órbita em torno da Terra, a força centrípeta é conservativa, pois é devida à interação gravitacional. No caso de um carro que faça uma curva em uma estrada, não é conservativa, pois é devida ao atrito dos pneus com o chão, bem como da normal exercida pelo chão sobre o carro, se a pista for inclinada para o lado de dentro da curva. Normal e atrito não são forças conservativas, do mesmo modo que tração e compressão.

suponha um objeto que esteja parado em uma altura igual a 0, sendo sua energia mecânica igual a 0. Então, uma pessoa ergue esse objeto até uma altura h, onde a energia mecânica será igual a mgh. Como a energia mecânica variou no trajeto, então a força que a pessoa fez não é conservativa?


Exatamente. Há três relações entre trabalho e energia na mecânica (sem envolver questões térmicas nem radiantes):
Variação de energia cinética = trabalho da força resultante;
Variação da energia potencial = - (trabalho das forças conservativas);
Variação da energia mecânica = trabalho das forças não conservativas.

Ernesto, sou muito questionador, acho que isso pode incomodar muita gente. Como lidar com isso?

É só não ligar para incomodar e continuar incomodando. Ser questionador é muito bom e é preciso que mais pessoas o sejam e que, de fato, incomodem muita gente.

Não entendi por quê não integrou o o r ^2 na expressão: V=π∫[r; -r] (r^2-x^2) dx V=π [r^2 x - x^3/3] r; -r Vi isso em um livro de cálculo.E não me venha com outra expressão, pois eu quero entender o porquê dessa!

Ora, porque a integral está sendo feita na variável x e não em r. Para essa integral, r é constante, como de fato é, pois é o raio da esfera.

Ernesto. Ontem, um autor me disse "Cara, pessoas inteligentes em 99% dos casos são de esquerda. Muda essa cabeça." Pelo fato de eu ser de direita. Vc corrobora com a afirmação dele?

De modo nenhum. Sou esquerdista mas posso ver que há burros e inteligentes tanto esquerdistas quanto direitistas. Admiro direitistas inteligentes, como foi o Simonsen, o Roberto Campos, o Delfin Neto, mesmo discordando de suas posições.

Você sente um grande amor pelo mundo e pelas pessoas em geral? Às vezes, tenho uma súbita sensação assim, acho que é quando a sertralina faz efeito

Sim. Mas isso não tem nada a ver com sertralina. Trata-se de uma cosmovisão pessoal, construída pelo que a pessoa já viveu e como ela interpretou o que o mundo lhe revelou. E isso não tem nada a ver, também, com alguma concepção religiosa. O que os religiosos fazem é inserir em suas doutrinas essa capacidade humana e transformá-la em prescrição. Como a do Cristo: "amai-vos uns aos outros". Mas isso não é exclusividade do cristianismo. Em geral é um preceito de todas as religiões e, mesmo, de uma moral inteiramente laica. Todavia não vai acontecer porque seja prescrito, mas porque seja algo que cada um sinta em seu íntimo, de modo involuntário.

Qual considera mais importante: o físico ou o engenheiro?

Ambos, claro. Um depende do outro em um processo de retroalimentação. E a sociedade depende igualmente de ambos. O engenheiro não é capaz de inventar nada que não seja baseado no que o físico descobre e o físico requer o engenheiro para construir os aparatos que usa em suas descobertas.

as leis em física são proposições? mas proposição não tem que ter um sujeito e um atributo? quanto fala que todo corpo em movimento com velocidade constante tende a permanecer a velocidade constante não é uma proposição. Então leis não são proposições. Diferena entre proposição e juízo?

Proposição é a expressão linguística de um juízo, que é um pensamento não verbalizado. As leis física são proposições sim, bem como os teoremas e os princípios. Essa lei da inércia que você mencionou, claramente é uma proposição, expressa por uma frase com sujeito, predicado, complemento e adjuntos.

Ernesto, a formação do sistema nervoso não se dá num momento único, é algo continuado. Então, qual seria o "marco zero"? E como saberemos se todo mundo tem o desenvolvimento igual, inclusive em dias, horas, minutos, segundos? Como estaria isso na lei? A ciência não sabe PORRA NENHUMA! Entendeu?

Mas eu não disse isso. Claro que a formação é gradativa. Contudo só a partir de dado momento é que se pode ter uma situação em que o organismo se torna senciente e, depois, consciente. A partir de então é que se pode dizer que aquele conjunto de células se trata de uma pessoa. A ciência não sabe tudo, mas sabe muita coisa. Muita mesmo. Muito mais do que as mitologias religiosas, que propõem descrições e explicações baseadas em palpites de quem as exarou.

Por quê a expressão (a^2 - x^2)= a^2 . Sen^2(teta) Fica : a^2- x^2= a^2. (1-sen ^2(teta)) Não entendi o porquê fo a^2 ter sido colocado em evidência, não era pra eles se cancelarem? E por que aparece o 1?

Essa expressão está errada.
a² = x² + y²
y² = a² - x²
y = a senθ
y² = a²sen²θ
a² = x² + a²sen²θ
a² - x² = a² sen²θ
sen²θ + cos²θ = 1
sen²θ = 1 - cos²θ
a² - x² = a²(1 - cos²θ)
Não tem como cancelar o a², pois ele não pode ser colocado em evidência no primeiro membro.

A questão não é a consciência, mas sim a vida. É uma vida humana em desenvolvimento, será que é realmente válido interrompe-la? Salvo em caso de estupro e risco de vida. Já disse e reitero: estupro é o crime mais nojento que existe.

E por que em caso de estupro? Se estupro é um crime, por que a criança gerada teria que pagar por ele se não o cometeu? Não estou dizendo que não se possa abortar em caso de estupro ou não. O que estou dizendo é que estupro não é uma razão diferente de outras. E que, se não se pode abortar, não se pode em caso de estupro também. E se se pode em caso de estupro e se considera que um embrião ou feto já seja uma pessoa desde a concepção, então o aborto, mesmo no caso de estupro, é um assassinato. Leia minha opinião a respeito nestes artigos:
http://wolfedler.blogspot.com.br/search?q=aborto

Popper dirá que o critério de demarcação entre uma ciência e uma pseudociência é a noção de refutabilidade. Enquanto a ciência é passível de testes e refutações, a pseudociência não o é. O que o senhor acha? Qual seria a sua demarcação para distinguir uma da outra?

Esse é um dos critérios que, realmente, diferencia a ciência da pseudociência. Outro é que os conhecimentos científicos são obtidos de uma forma criteriosa e metódica, enquanto as pseudociências se funda em proposições aventadas por palpites de quem as exara. A verdadeira ciência não se baseia em opiniões não justificadas nem testadas.

"aproximação cada vez maior da verdade" Sim, mas dessa vez não é para descobrir se buracos de minhoca são ou não reais. De duas, uma: Ou você acha que ciência não erra, ou você acha que a ciência PODE ERRAR e, portanto, considera justo matar um ser humano em caso hipotético. Escolha.

Não consigo ver nenhuma correlação entre achar que a ciência não erra ou que pode errar e o fato de se considerar justo ou injusto se matar um ser humano. Eu acho que a ciência pode errar e acho que não é justo matar um ser humano, exceto em legítima defesa.

https://goo.gl/c3KQe7 ... Quem disse a você que a ciência, hoje, tem a capacidade de definir o que é consciência, de onde e quando ela surge? A deusa "ciência-moderna" é uma espécie de criança mimada, que se contradiz o tempo todo e, quando erra, se diz inimputável por ser criança imatura.

O grande mérito da ciência é, justamente, não ser definitiva e sempre estar em busca de uma aproximação cada vez maior da verdade. Se não se tiver isso não se tem nada. Dizer que a ciência vai se corrigindo à medida que conhecimentos vão se acumulando não é desfazer dela, pelo contrário, é reconhecer seu valor. A ciência não se contradiz, ela se corrige, à medida que vai aprendendo. Isso é que é o correto e desejável. Quanto à consciência, trata-se de um dos maiores temas de pesquisa atuais. Mesmo que ainda não se tenha um quadro definitivo (e pode ser que jamais se o tenha), muito já se tem, dentre o que o fato de que a consciência advém do funcionamento do sistema nervoso. Portanto, o embrião e o feto, antes que tenha um sistema nervoso, não tem consciência. Isso é tranquilo. Atribuir a consciência a uma pretensa "alma" existente é que é algo inteiramente despropositado.

eV é uma unidade de energia bem baixa se comparado às energias cotidianas dos processos macroscópicos. Por que, então, é tão difícil prover energias da ordem de J ou kJ para partículas subatômicas em experimentos? Qual a limitação experimental?

Se uma partícula elementar alcançar uma energia da ordem de joules, isso será da ordem de quintilhões de eletrons-volts, ou seja exa-elétron-volt (EeV). Os maiores aceleradores já construídos conseguem fornecer energias à partículas da ordem de dezenas de tera-elétrons-volts (10TeV), que é um valor cem mil vezes menor. Um acelerador para EeV teria que ter um comprimento cem mil vezes maior do que o LHC, isto é, 70 vezes maior do que o equador da Terra ou uns sete décimos da distância entre a Terra e a Lua. Isso se o campo acelerador fosse o mesmo.

Mais um episódio da esquerda "paz e amor", que defende a pluralidade de ideias: https://www.facebook.com/mblivre/videos/620868698037277/

Esquerdistas que não aceitem que se possa ter opinião divergente deles prestam um enorme desserviço à causa da esquerda e, em verdade, não são verdadeiramente de esquerda, pois esquerda é uma concepção política, econômica e social que defende a liberdade. Esquerdista coerente não pode ser a favor de regimes ditatoriais e nem ter qualquer opinião policialesca em relação a quem não seja de esquerda. Senão estará cometendo o mesmo tipo de erro que as ditaduras de direita cometeram e cometem em não aceitar quem seja de esquerda.

https://ask.fm/wolfedler/answers/141424280605 ... E criminalizar o aborto? "Ah, não! A vida de uma ovelha vale mais!" Psicopata! Vou procurar na Deep Web um aborto filmado. Instale o Tor Browser aí. Imagine um dispositivo de sucção violento... É para assistir, hein, psicopata?

Não vi em que minha resposta mencionada diga algo a respeito da criminalização ou não do aborto. Nem que eu tenha dito que a vida de uma ovelha valha mais do que a de uma criança. Não vou ver nada na Deep Web porque abomino essa instância. Quanto ao aborto, veja o que eu já disse a respeito:
http://wolfedler.blogspot.com.br/search?q=aborto

Preferias viver para sempre ou viver 10 vezes durante 100 anos?

Para sempre é impossível, pois o Universo, em qualquer lugar, se tornará hostil a qualquer tipo de vida em alguns trilhões de anos. Mas eu gostaria de viver indefinidamente, enquanto houvesse condições ambientais para o suporte da vida.

Ernesto, duas perguntas. Por que não é utilizado o formalismo Lagrangeano na Mecanica quantica? Por que é necessário utilizar números complexos na MQ?

Lagrangeano não, mas hamiltoniano sim. A mecânica quântica é construída a partir da mecânica clássica hamiltoniana pela substituição dos parêntesis de Poisson pelos comuntadores dos operadores associados às grandezas.
https://pt.wikipedia.org/wiki/Representa%C3%A7%C3%A3o_de_Heisenberg

Você já sofreu preconceito por ser ateu?

Não. Nunca. Talvez porque o meio em que vivo seja de pessoas intelectualizadas. Por outro lado, percebo que as pessoas, mesmo as que são religiosas, vêm que meu ateísmo é uma convicção sincera e que sou uma pessoa de bem.

RESUMO DA DEFESA SOBRE O ATEÍSMO: ''EU ACHO'', ''PODE SER'', ESSE É SEU ARGUMENTO PARA DEFENDER A NÃO EXISTÊNCIA DE ALGO? ACHISMOS BARATOS? PELO QUE LI NUNCA FOI NA ''CASA'' DAS RELIGIÕES PARA COMPROVAR NADA, APENAS DEDUZ E OPINA SOBRE, QUE BRILHANTE MÉTODO DE ESTUDO. (IRONIA)

Sim. Eu acho mesmo. É o que eu acho. E eu pauto minha vida pelo que eu acho. Não tenho certeza de que deuses não existem, como não se tem certeza de que existam. Quanto a estudar religiões, foi justamente isso que me levou a ser ateu. Sem comprovações mesmo. Mas com fortes indícios.

NÃO SEJA LEVIANO, MEU CARO PASTOR, O VINHO BRANCO QUE CONSOME TEM INGREDIENTES DE ORIGEM ANIMAL, QUEIJOS, CERVEJA, GELATINA, IOGURTE, ETC...PESQUISE SOBRE O QUE CONSOME, ANTES DE SE ROTULAR VEGETARIANO E DIZER QUE É CONTRA ASSASSINATO DE ANIMAIS, JÁ QUE CONTRIBUÍ PARA ISSO DESCARADAMENTE.

Sou vegetariano e não vegano (mas posso me tornar). Laticínios não requerem abate de animais para serem feitos. E vinho não tem componente animal não.

http://sensoincomum.org/2017/06/21/rodrigo-hilbert-matar-ovelhas ... A próxima meta esquerdista é criminalizar o consumo de carne. Esquerdista mora num mundo abstrato de ideais, não vive na realidade. Dinheiro é feito em árvores; carne, em açougues. E assim por diante...

O vegetarianismo e o vaganismo são concepções que não têm nada a ver com direita e esquerda políticas e econômicas. Há pessoas vegetarianas e veganas em ambas as concepções politico-econômicas. E o próprio vegetarianismo e o vaganismo podem ser por razões nutricionais ou razões éticas. Sou um vegetariano por razões éticas. Para mim o assassinato de animais para consumo deveria ser um crime mesmo. Mas isso não tem nada a ver com minhas concepções esquerdistas, por ser um anarco-comunista (totalmente infenso ao socialismo de estado, o capitalismo e o capitalismo de estado). Sua ideia de que esquerdistas não vivem na realidade não é correta. Eles vivem na realidade e intentam modificá-la para se tornar uma realidade boa. Transformar a realidade social é o objetivo da esquerda. Alguns, equivocadamente, consideram que isso possa se dar pela "ditadura do proletariado", implantada por uma revolução. Mas revoluções não resolvem. O comunismo anárquico será estabelecido no mundo por meio da evolução civilizatória da humanidade, ao fim de séculos de transformações, até que o dinheiro, a propriedade, o estado e o governo deixem de existir por absoluta falta de necessidade de que existam.

O que te faz se sentir livre?

O fato de ser livre, ora. Posso ir para onde quiser e conseguir. Posso fazer o que quiser e conseguir, desde que não prejudique a ninguém. Posso pensar o que quiser. Posso dizer o que quiser. Posso ser o que quiser ser, exceto ser criminoso. Então sou livre. Se não consigo algo não é porque não tenha a liberdade de conseguir e sim porque não disponho de condições para conseguir, de que tipo for. Felizmente vivo em um lugar que, apesar de muitas mazelas, é um lugar livre. Isso é ótimo.

Professor sobre cursar fisica, Compensa ter um conhecimento teórico pra viver com recursos razoáveis? Eu dou valor a isso, entao limita minha escolha.. Nos eua tem espaço pra físicos, emigrar como seu filho fez?

Sim. Normalmente o que acontece é que a pessoa vai estudar lá, por exemplo o doutorado e se mostra muito bom de serviço, então eles mesmos convidam para ficar trabalhando lá. Eles dão muito valor às pessoas realmente competentes, sejam de que país forem.

FIZ ALGUMAS PERGUNTAS PARA VER A SUA CAPACIDADE, VOCÊ CLARAMENTE NÃO RESPONDEU NADA, APENAS JUSTIFICOU A SUA FALTA DE INFORMAÇÃO SOBRE O ASSUNTO, FALOU , FALOU E NÃO DISSE NADA, NÃO FAÇO PARTE DO SEU GADO,PASTOR, EU SEI QUANDO VOCÊ ESTA ERRADO E QUANDO TEM LIMITAÇÃO EM DETERMINADAS ÁREAS.

Publicado para conhecimento de sua opinião. Não sou pastor de nada. Como não respondi? Respondi tudo que você me perguntou.

O ''NADA'' É A RESPOSTA PARA TUDO, ESSA É A DEFINIÇÃO DE TUDO QUE EXISTE, SURGIU DO NADA, ACABARA NO NADA, TODO ''NADA'' TEM UM COMEÇO E UM FIM, POIS SEM FIM NÃO SERIA ''NADA''.

Nada disso. Para começar, não é "o nada" e sim, "nada", sem "o". Se tiver artigo está se referindo a um substantivo que representa uma entidade. "Nada" é um pronome indefinido que se reporta à ausência do que quer que seja. "Nada" não é uma entidade, não é algo. Portanto "nada" não tem existência e, assim, não tem começo e nem tem fim. Nem extensão nem localização. Nada é "nada". Se "é" algo, não é "nada".

A MENTE É CAPAZ DE ENXERGAR O ABSOLUTO? É CAPAZ DE CONCEBER O NADA?

O que vem a ser o "absoluto"? Quanto a"o nada", é claro que a mente não percebe como seja, pois é algo que nem existe nem é concebível que exista. Todavia pode-se conceber que não exista nada, o que é muito diferente de conceber que exista "o nada". Se você encostar o polegar no indicador, supondo que não existam os sulcos digitais, o que fica entre eles? Nada! Conteúdo nenhum e sequer espaço vazio para poder conter algo. Isso é "nada". Mas não é "o nada", pois não se trata de algo.

DE ONDE VEM O QUE EXISTE?

De nada. Surgiu espontaneamente, sem ter de que provir, sem ter causa para ter surgido e sem ter propósito para isso.

QUAL A POSSIBILIDADE DE ALGO TER SEMPRE EXISTIDO?

Não só algo, mas todo conteúdo substancial do Universo, mesmo que não estruturado nas entidades atualmente existentes (galáxias, nebulosas, estrelas, planetas), pode, perfeitamente, ter existido desde sempre. Todavia o que se verifica é que não é assim. Tal conteúdo, bem como o espaço que ele ocupa e o tempo em que evolve tiveram um início em um momento zero, do qual não existe "antes".

SE ANTES NÃO HAVIA NADA, DO QUE ERA FEITO?

Ora, por definição, se não há nada, não é feito de nada. Nada não é algo. É a designação da ausência de tudo. Conteúdo (campo, matéria e radiação), bem como espaço e tempo (e, consequentemente, estruturas e ocorrências). Inclusive não havia "antes". Por isso não existe "o nada", ou seja, não existir nada não significa existência d"o nada".

ENQUANTO HÁ PESSOAS PREOCUPADAS EM CONVENCER OS OUTROS DA INEXISTÊNCIA DE DEUS, EXISTEM PESSOAS FOCADAS EM PROCURAR A VERDADE, E ESSA VERDADE É OBSCURA, MUITO PIOR DO QUE IMAGINAM. PROCURE SABER A VERDADE MEU CARO CONTADOR DE ESTÓRIAS.

Isso é, exatamente, minha busca. E a inexistência de deuses é uma das que já verifiquei ser correta. Portanto, sinto-me na obrigação de proclamá-la para que, justamente, as pessoas não continuem na ignorância a respeito.

SE VOCÊ TIVESSE SABEDORIA COM CERTEZA NÃO ME QUESTIONARIA!

Acho isso muita pretensão de sua parte.

HÁ UMA BASE MILITAR NO DESERTO ONDE A EVOLUÇÃO REAL DA TECNOLOGIA É USADA E EXPLORADA, QUASE SEMPRE PARA FINS MILITARES, AS INFORMAÇÕES QUE SABES SOBRE UNIVERSO PODEM SER REAIS, PORÉM, VOCÊ POSSUI 0,001% DO QUE ELES PERMITEM.

De onde você tirou esse número? Você faz parte dessa equipe? Se não faz, como você sabe o que eles sabem ou não sabem para dizer que o que eu sei seja um décimo milésimo do que seja permitido saber? E o que é permitido saber? Que fração é do que se sabe (isto é, do que eles sabem)? Como saber? E como saber que isso que você está dizendo é verdade?

''SE VOCÊ É BOM EM ALGUMA COISA, NUNCA FAÇA DE GRAÇA''

De graça é como tudo deve ser feito em uma sociedade ideal em que a economia é uma economia de doação e não existe nem dinheiro nem propriedade.

COLOCAI UM OBJETO NO ESPAÇO, SUA PRESENÇA SE TORNA INTRUSA, LOGO O UNIVERSO SE ''DEFENDE'' E AGE DE FORMA INCONSTANTE E IRREAL, NADA PODE SER VISTO A OLHO NU. AS ILUSÕES DOMINAM A MENTE FERTIL E DOENTIA QUE BUSCA RESPOSTAS PARA O TUDO, EMBORA SEMPRE SE ENCONTRE O NADA.

Nada disso! Qualquer objeto colocado em qualquer lugar é simplesmente algo que foi tirado de outro lugar. Portanto não é um intruso. E o Universo não se "defende" de nada. Buscar respostas não é nada doentio, pelo contrário, é extremamente saudável. E "o nada" jamais é encontrado, pois não existe.

BURACOS NEGROS SÃO ATALHOS PARA OUTROS LUGARES NO UNIVERSO E NÃO LEVA A UM ''UNIVERSO PARALELO''. AS VEZES USADOS COMO LIXEIRA UNIVERSAL, ELIMINANDO CORPOS ESTRANHOS, HÁ UM BURACO NEGRO A CADA 100.000 KM.

Isso é uma conjectura não confirmada.

INTELIGENCIA NÃO SIGNIFICA MUITA COISA, UMA PESSOA SE TORNA INTELIGENTE AO SABER SOBRE DETERMINADO ASSUNTO, SABEDORIA SIM É UM ELOGIO,AS VEZES UM DOM, SER INTELIGENTE ATÉ UM ANIMAL PODE SER, USAM DE FORMA ERRADA O TERMO, ASSOCIANDO-O A ALGUÉM SÁBIO.

Inteligência não é sapiência. Dominar muitos conhecimentos não significa ser inteligente. Inteligência é a capacidade de aprender sem dificuldade e de solucionar problemas de qualquer ordem. Certamente que a sabedoria é mais valiosa do que a inteligência, como tenho sempre dito, se você acompanhar o que digo.

SOBRE RECLAMAR SOBRE POLÍTICA, NÃO RECLAME, RESOLVA. SOBRE O QUE DIZ SOBRE LEIS, NÃO RECLAME, RESOLVA. SOBRE O QUE DIZ SOBRE DEUS, NÃO QUESTIONE, PROVE. QUERES QUESTIONAR TUDO, POLEMIZAR... POR QUE NÃO USA O SEU TEMPO PARA RESOLVE-LOS? SEU TEATRO NÃO ESTA MUDANDO O MUNDO COMO PENSA.

Questionar é importantíssimo, mesmo que não resolva de imediato. Mas conduz à solução futura. Quanto ao que digo sobre deuses, sobre economia e política ideais para o mundo, sempre argumento justificando minhas opiniões. Basta você acompanhar o que venho dizendo. Enquanto isso, vou fazendo o que posso para implementar minhas idéias, como, por exemplo, fazendo muito trabalho de graça e incentivando as pessoas a também trabalhar de graça. Não acho que o que eu faça seja um teatro, pois não estou representando o que não sou. Pelo contrário, sou autenticamente do modo como me apresento. Para finalizar, quero comentar que o uso de maiúsculas em todo o texto é altamente deselegante. E que a elegância é algo necessário, mesmo para contestar o que for. Seja sempre uma dama ou um cavalheiro.

Professor voce acha que no futuro a viagem para o futuro a bordo de uma nave voando a velocidade da luz sera possível? Hoje daria pra acontecer essa viagem? Fico imaginando se o Bill Gates resolve viver mais, sera que ele tem recursos para isso?

Uma viagem à velocidade da luz, para algo material, é completamente impossível. Mas é possível se conseguir a uma velocidade próxima da da luz. Todavia isso requereria um consumo imenso de energia, não acessível mesmo que se reúna toda a energia disponível na Terra. Além de ser uma proeza tecnológica só capaz de ser atingida num futuro de séculos.

Você é disciplinado?

Um pouco, mas nem tanto. Todavia me esforço por ser, pois acho que é bom. Só que não me apoquento quando não sou.

A pessoa realmente pode mudar hoje e ter um fim de vida melhor ou não podemos fugir do que sempre fomos?

Claro que pode. Qualquer um, a qualquer tempo. Basta ter decisão e força de vontade, pois é muito difícil. Ninguém é fadado a ser como sempre foi.

https://ask.fm/wolfedler/answers/141420454173 Por que, na sua concepção, a fusão a frio não é possível?

Porque para que os prótons se aproximem o suficiente para que sofram a força nuclear é preciso que possuam uma energia cinética muito alta, o que apenas consegue ser fornecido se eles estiverem a uma temperatura de milhões de kelvins. Outra alternativa é que eles sejam acelerados em aceleradores de partículas. Mas isso não é "Fusão a Frio".

Ernesto, Landau (Mechanics) define um referencial inercial como sendo aquele onde o espaço e o tempo são uniformes e isotropico. Essa definição é equivalente a que encontramos nos livros convencionais? Voltando a questão do tempo nao uniforme, esse só é observado próximo da velocidade da luz, né?

Sim, de fato, se o referencial não for inercial, há diferença em movimentos que se deem na direção de sua aceleração em relação a um que seja inercial e movimentos em outras direções. Mas essa conceituação é mais abstrata e menos inteligível do que a baseada nas Leis de Newton. Quanto à não uniformidade do tempo, ele é observada mesmo em movimentos a baixas velocidades, comparadas com a da luz:
http://www.fflch.usp.br/df/opessoa/TR-Exp-2-Avioes.pdf

Ao pesquisar seu nome no google aparecem processos com seu nome, sabia disso?

Sim. Trata-se de uma ação movida por um grupo de professores contra a Universidade Federal de Viçosa para que lhes fosse autorizado fazer concurso para Professor Adjunto sem ter o doutorado, de acordo com as disposições transitórias do Regime dos professores universitários que havia sido negado pelo Conselho Universitário.

O Colégio de Aplicação da UPE, uma renomada faculdade, tem desde o Quinto ano do ensino básico, alunos. Se os CAP'S fossem todos assim, talvez houvesse um progresso considerável no âmbito educacional.

O Colégio de Aplicação da Universidade Federal de Viçosa também é excelente. Acho que não há motivo nenhum para que toda escola não seja assim. É só uma questão de vontade e disposição para arrostar a oposição à qualidade que existe por parte de muita gente, inclusive alunos. É preciso que assim o seja e que o Ministério da Educação exija das Secretarias de Ensino dos Estados e dos Municípios que primem pela alta qualidade do ensino, mesmo que isso provoque reprovações em massa e represamento. Se se mantiver firme nessa exigência, em poucas décadas esse represamento acaba, pois os estudantes vão ter que conseguir aprovação a todo custo, por parte do esforço pessoal deles. Claro que, nesse afã, é preciso demitir todos os professores incompetentes e contratar outros bons no lugar. E para que os bons se interessem por ser professores, é preciso pagar bem. Bem mesmo. Poder-se-ia colocar o atual quadro em extinção e começar um novo, com ótimos salários e dedicação exclusiva, acessível por uma seleção rigorosa, ao qual todos os antigos professores poderiam acessar, se fossem aprovados na seleção. Se não fossem, ficavam no quadro em extinção com o salário baixo até se aposentarem. Ou seriam demitidos por incompetência, desde que se tirasse a estabilidade dos funcionários públicos, que é uma urgente necessidade nacional.

Tenho uma amiga que concorda com tudo o que digo, de modo tal que ter uma amizade com ela passou a ser desinteressante para mim. Gosto de pessoas que sejam capazes de discordar também e tenham opiniões próprias. Por acaso já desinteressou de algum amigo que sempre concorda, em tudo, com você?

Ainda não tive nenhum amigo ou amiga assim porque, em geral, minhas concepções são muito polêmicas e, dificilmente, acho alguém que concorde comigo em tudo.

Ernesto. Afinal de contas a fusão a frio é possível ou não? Quais seriam os benefícios para a humanidade e a natureza se algum dia pudermos produzir energia através da fusão a frio?

Entendo que não seja possível. Se o fosse, seria ótimo, pois os reatores de fusão têm um rendimento térmico maior, além de usarem um combustível mais barato: água.

Ainda vale a pena estudar taquigrafia ou esta é uma profissão que deixará de existir?

Só se for por curiosidade. Porque profissionalmente a taquigrafia está fadada a ser extinta.



Hoje em dia, aumentaram o número de cursos nas Faculdades. Quais os cursos "em alta" na Universidade ou que têm maior campo de atuação? E os cursos em baixa, os quais aconselharia as pessoas não fazerem?

Não aconselho ninguém a deixar de fazer qualquer curso, porque a pessoa tem que fazer aquilo que gosta, independentemente do fato da profissão do curso ser ou não rentável. Ou mesmo que não faça curso superior nenhum, o que, absolutamente, não é preciso, nem para ser rico e nem para ser feliz. Infelizmente, no Brasil, só existem cursos superiores correspondentes a profissões. Deveriam haver cursos apenas para aprimoramento cultural, científico, filosófico, artístico, ou empresarial, sem dar profissão nenhuma. A pessoa auferiria renda por meio de qualquer ocupação. Ela poderia, por exemplo, ser uma herdeira de muitos imóveis e viver da renda de seus aluguéis, não precisando ter profissão nenhuma. Ou ser uma vendedora que queira ter um aprimoramento cultural sem nenhuma ligação com seu trabalho. Ou uma capitalista que viva do rendimento de suas ações.

Professor o que acha sobre os famosos casos de ufologia aqui no brasil, como o de varginha? Varias e varias evidências

Tudo isso é uma total empulhação. Nada verídico.

Uma sociedade virtuosa não seria uma sociedade verdadeiramente católica? A hagiografia prova isso. Me diga o nome de um único ser humano, de qualquer época, que tenha sido mais virtuoso que qualquer cristão canonizado.

Não. Porque seria? Uma sociedade católica pode ser virtuosa, mas uma sociedade budista, islâmica, hinduísta, zoroastrista ou protestante também pode ser. Do mesmo modo que há pessoas muçulmanas, budistas, protestantes, hinduístas ou zoroastristas que não sejam virtuosas, também há pessoas católicas que não o são. Não é o catolicismo que vai garantir a virtude da sociedade. Como também pessoas ateias podem ser muito virtuosas. Ou não. Do mesmo modo que houve cristão canonizados, também houve muitas pessoas das outras religiões que, mesmo não tendo sido canonizadas, foram extremamente virtuosas.

Condensado de Bose-Einstein

https://pt.wikipedia.org/wiki/Condensado_de_Bose-Einstein
https://en.wikipedia.org/wiki/Bose%E2%80%93Einstein_condensate

É crime ter um perfil anônimo na internet? Mesmo se for um perfil com uma foto de um animal e como nome Koala Fofo ou Golfinho Carinhoso e uma imagem de domínio público desses respectivos animais? Supondo que a pessoa de fato não queira se expor por cautela, mas queira participar de redes sociais?

Claro que não é crime. Nem é errado. Desde que a pessoa não use esse anonimato para difamar, caluniar, espezinhar, fofocar, pregar preconceitos, ser intolerante (exceto com a maldade). No mais ela pode até fazer proselitismo de suas ideias, sejam quais forem, desde que não nefastas, como pregar o sacrifício ritual de criancinhas em homenagem a alguma divindade.

Professor estou pensando em cursar ciência da computação, vi que voce preza pela liberdade de criacao, como um artista, um musico, e tambem gosto de exatas, se tudo der errado, terei adquirido um raciocinio bem logico que vale muito, entao acho que é um bom caminho, o que voce acha?

Ciência da Computação é bem interessante, mas Física é muito mais ainda. Além de que a Física é a ciência mais filosófica e menos prática, o que acaba levando físicos a filosofar mais e a querer conhecer mais filosofia. Por outro lado há mais físicos músicos, pintores, poetas, literatos do que outros cientistas. Todavia, Ciência da Computação já é uma atividade científica que requer bons conhecimentos de matemática e lógica. Melhor seria Engenharia da Computação, que envolve mais Física também, além de abranger quase tudo o que a Ciência da Computação abrange.

Segundo os esquerdistas, quem não concorda com as asneiras que eles defendem é fascista, nazista, racista, eletricista, taxista...

Isso é um grande equívoco da parte deles. Do mesmo modo que os direitistas acham que quem não concorde com tudo o que eles defendem é marxista. Ora, uma pessoa de bom senso tem que examinar tudo e concordar em parte com a direita, em parte com a esquerda, naquilo que uma ou outra tenham de correto e discordar de uma e da outra no que possuam de errado. Da mesma forma que pode concordar em parte com o cristianismo, em parte com o ateísmo, em parte com o islamismo, em parte com o budismo, em parte com a democracia, em parte com a anarquia, em parte com o socialismo, em parte com o capitalismo, em parte com o comunismo e assim por diante.

Sinto constantemente que não estou sendo eu mesmo quando estou em ambientes sociais, quando estou em contato com outras pessoas. Já se sentiu assim?

Não. Mas isso é possível. Todavia você pode continuar a ser você mesmo, estando com alguém ou em ambiente social. Basta não fingir ser o que não é, mesmo que isso seja estranho para os outros ou provoque um afastamento das pessoas de você. Mas, pelo menos, você permanece autêntico. Não é preciso se fazer de agradável e satisfeito se não se está. É só não ser mal-educado.

Há evento pelo universo que podem destruir nosso planeta sem dar/ou quase aviso prévio?

Sim, da mesma forma que o Universo surgiu, sem provir de nada, ele pode, de repente, desaparecer inteiramente, se deixar nada no lugar, sequer espaço vazio.

Ernesto, o que você acha do sistema universitário brasileiro, com um organograma completo para o aluno estudar do inicio ao fim, não deixando ele guiar a formação p/ a carreira que ele quer seguir? Você acha que o sistema dos EUA é melhor ou não?

Pelo que me consta não é assim não, pelo menos na UFV. Cada curso possui um elenco de disciplinas obrigatórias (cerca de três quintos), complementadas por disciplinas optativas escolhidas em uma lista bem grande. Além dessas o estudante pode cursar qualquer disciplina da Universidade, desde que tenha feito seus pré-requisitos. Só que, matriculando-se, precisa ser aprovado para levar o diploma. Veja, por exemplo, o currículo do Curso de Física:

esse caso do serial killer Andrei Chikatilo, de ter matado cruelmente várias crianças e inclusive, canibalizado elas, cruelmente, é uma prova de que deus não existe, visto que provavelmente ele era um doente?

O fato de ser uma pessoa extremamente má e cruel não comprova a inexistência de Deus, em absoluto. E isso, inclusive, também não é indicativo de que a pessoa seja portadora de alguma doença mental. Uma pessoa perfeitamente sã pode ser extremamente malvada e cruel.

Tive uma palestra com o prof Suntzeff (desobridor da energia escura) e ele disse que a cosmologia pode deixar de existir (em bilhões de anos), uma vez que a inflação pode se dar em "velocidades" superiores às da luz e a informação de eventos na borda da inflação jamais chegaria até nós. Como?

Isso já acontece atualmente. Parte do Universo já se encontra além do que seja possível observar porque se afasta de nós mais rápido do que a luz que, então, não consegue nos atingir. À medida que o tempo passa, mais porções do Universo passam para fora dessa fronteira. No futuro não se terá acesso a nada além de nossa própria galáxia. Mais tarde ainda, nada além do Sistema Solar. Mais tarde ainda, a própria eletrosfera de um átomo pode estar mais longe do núcleo do que essa fronteira. Isso, pelo que se sabe, não vai parar nunca.

Quais são as possíveis falhas ou fragilidades do método cientifico ?

O método científico é um processo de tentativas e pode ser que a ciência ainda não disponha de competência suficiente para proceder a alguma investigação do comportamento da realidade em algum aspecto. Essa é a sua deficiência que, todavia, aos poucos vai sendo contornada, com o desenvolvimento de técnicas de pesquisa cada vez mais elaboradas. Assim a ciência vai progredindo paulatinamente e, dia a dia, consegue explicar mais coisas que antes não conseguia.

Errado pela lei e pelo ponto de vista de alguns, quando a pessoa faz algo pelo seu próprio bem e pelo bem dos outros não é crime.

É sim. Porque crime é algo que se faça em desobediência à lei. Todavia, se a lei for pérfida, há que se desobedecê-la e, assim, cometer o crime, em prol do bem. Certamente assumindo todas as consequências disso, inclusive a prisão ou a condenação à morte, quando existir. Morrer em defesa do bem é glorioso.

Por que quando se aplica a equação reduzida da circunferência (x-a)^2+(y-b)^2 sobre a esfera, ficaria o a=0 b=0? Por que isso acontece?

Não. Só se a esfera estiver com seu centro na origem.

Se eu somar (-cos^2 + cos^2) os termos vão se anular? Já que terão coeficiente zero

Sim, se o quadrado não abranger o menos, ou seja, se não for (-cos)^2 + (cos)^2. Neste caso o resultado é 2(cos)^2, já que (-cos)^2 é diferente de -(cos)^2.

https://www.facebook.com/DefesaDaTradicao/?hc_ref=NEWSFEED&fref=nf olhe o conteúdo dessa página, ridículo!

Tradição não tem valor apenas por ser tradição. Dependendo do assunto ela pode ser boa ou ruim. Se contribuir para o bem geral é boa. Mas, mesmo sendo boa, não pode ser adotada de forma rígida. É preciso se aceitar que seja contrariada se isso também for algo bom. O tradicionalismo conservador e católico que esse perfil defende não é bom por ser intransigente em suas concepções. Da mesma forma que o tradicionalismo islâmico. Por outro lado, nem tudo que seja novidade, por assim o ser, é algo bom. Há novidades boas e ruins. Não aceito o fundamentalismo islâmico e nem aceito o fundamentalismo cristão. O que aceito é uma concepção aberta e livre pensadora, que pode concordar quer com o cristianismo, quer com o islamismo, quer com o capitalismo, quer com o comunismo, naquilo que um ou outro deles for bom e discordar naquilo que um ou outro deles for ruim.

Você acha que no assalto em que Bin Laden foi morto, deveriam ter usado armas não letais e feito de tudo para preservar a vida dele?

Acho. Matar é errado em qualquer circunstância, exceto em legítima defesa, no momento do ataque.

https://pt.wikipedia.org/wiki/Andrei_Chikatilo uma pessoa dessa, deveria ser morta ou apenas prisão perpétua? o que acha dele?

Não importa o quão cruel um assassino seja, nem quantos tenha matado, matá-lo, mesmo que legalmente, é um assassinato também e é errado. Mas ele merece prisão perpétua com trabalhos forçados.

http://epocanegocios.globo.com/Brasil/noticia/2017/06/expandiu-se-demais-investigacao-alem-dos-limites-diz-gilmar-sobre-lava-jato.html será que este cidadão está com medo de que as investigações se aprofundem mais? Por que será?

Não pode haver limite nenhum nas investigações. Elas são para colocar medo nas pessoas sim. Quem não deve não teme. Quem teve tem que temer e muito. Todas as falcatruas de políticos, empresários, juízes ou seja quem for têm que ser investigadas a fundo. E os culpados punidos exemplarmente, com a perda dos direitos políticos por toda a vida e o confisco dos bens até que devolvam tudo o que roubaram, além da prisão fechada pelo tempo todo da pena, sem nenhum alívio, em penitenciárias de presos de crimes comuns e do narcotráfico. Naquelas celas para dez pessoas em que ficam cem.

Olá, Professor! Tudo é exato em ciências exatas? Ou as vezes ocorre de algo não ser exato, como é mais comum em Humanos ou Biológicas...

Não. Em ciências exatas a maior parte do que se estuda não é exata e sim aproximada.

professor , você tem alguma dica de como se tornar tão inteligente como você? que livros você indica para iniciantes em física?

Primeiro estude de novo a Física do Nível Médio em um bom livro, como o PSSC (Physical Sciences Study Committee), incluindo os tópicos avançados (https://pt.scribd.com/doc/294531112/PSSC-TopicosAvancados-Cap01). Ou o Jay Orear. Depois passe para um bom livro de Física Básica de nível superior, como o Curso de Física de Berkeley em cinco volumes ou o Alonso & Finn, que agora tem em volume único, editado em Portugal. Para ser bem inteligente, além de ter sido dotado de uma capacidade nata nesse sentido, é bom praticar exercícios neuróbicos. Procure por isso na internet. Veja, também, os livros do professor Pierluigi Piazzi sobre inteligência.

Professor, com o avanço da sua idade, tem sentido alguma dificuldade no aprendizado de novos assuntos, lhe parece mais difícil aprender algo novo do que quando tinha entre 15 e 30 anos de idade? Aprender um novo idioma é impossível?

Não. Nem um pouco. Minha capacidade de aprendizado continua intacta, para qualquer assunto, inclusive novos idiomas. Ainda não percebi nenhuma redução nem da capacidade de aprendizagem nem da capacidade de raciocínio e de imaginação. Do mesmo modo que a capacidade de se emocionar e ter sentimentos, bem como de ter força de vontade.

Professor, o pensamento racional, exato e logico de quem faz engenharia ou algum curso de exatas é vero? Pra quem quer mexer com negócios o senhor acredita que pode te dar alguma vantagem? Ou uma formação mista como administracao pode ser melhor para esse fim? Obrigado

Vamos entender que o que você quer dizer por pensamento seja raciocínio (porque há pensamentos que não são raciocínios, como emoções e juízos). Veracidade não é uma propriedade dos raciocínios e sim de suas consequências. A qualidade desejável de um raciocínio é sua validade, que significa o fato de levar a conclusões corretas a partir das premissas admitidas. Se, contudo, as premissas forem falsas, mesmo que o raciocínio seja válido, as conclusões não serão verdadeiras, o que também pode acontecer, mesmo que as premissas sejam verdadeiras, se o raciocínio não for válido. Quanto à exatidão, como já comentei na resposta sobre medidas, trata-se de uma qualidade quase impossível de se obter, devendo-se se contentar com a precisão. Realmente o treinamento que os profissionais das carreiras exatas fazem também é muito útil na administração de negócios. Tanto que há muitos economistas, especialmente os mais antigos, que são engenheiros. Todavia o domínio de conhecimentos específicos de administração e economia é muito importante para tal. Uma formação em economia, administração e engenharia é o ideal para uma pessoa que vá gerenciar negócios. Todavia há que se ter, também, um pendor e um tino especial, ou seja, um dom nato, para tal atividade.

pode-se afirmar que se mediu algo e está 100% exato ou isso não existe?

A exatidão em medidas é algo praticamente impossível de se obter. Só se consegue em contagens. O que pode se ter é a precisão, que significa que várias medidas da mesma grandeza apresentam um desvio absoluto médio ou o desvio padrão (raiz quadrada positiva da média dos quadrados dos desvios), em relação à média, pequeno. O verdadeiro valor da grandeza é desconhecido e se toma a média de várias medidas, feitas com o máximo de acurácia como se fosse o valor verdadeiro.

Professor, tornar um ponto referencial é fazer com que ele fique com velocidade = 0, em relação ao planeta Terra?

Não. A velocidade de um referencial pode ser qualquer uma, quer em relação à Terra, quer em relação ao que for, como o Sol, a Via Láctea, um conjunto de Galáxias. Inclusive não precisa ser uma velocidade constante.

Voce descarta completamente a vida apos a morte (espirito)?

Completamente. Vida é só a vida biológica de um organismo. Com a cessação de seu funcionamento, a mente daquele ser cessa de existir. A mente é decorrente da anatomia e fisiologia do sistema nervoso dos animais.

Esses dias dei uma lida sobre os teoremas da incompletude de Gödel. Se eu entendi bem, dado o segundo teorema, não é possível provar se a matemática é consistente (por consequencia boa parte da [ou toda a] ciência). Você tem fé que a matemática é consistente?

Os teoremas de Gödel não se referem à matemática e sim à aritmética, que é uma parte dela. E não se referem às demais ciências no que elas não dependem da aritmética. Considerar que a aritmética seja ou não consistente não é uma questão de fé e sim de verificação. Se se aceitar os axiomas em que ela é baseada, ela é consistente. Só que eles não podem ser provados, o que não contraria os teoremas de Gödel.

Professor, o Sol visto fora da atmosfera terrestre tem coloração branca, certo? Ou seja, é a atmosfera a responsável por espalhar a luz e consequentemente fazer com que enxerguemos o Sol amarelado?

Sim, é isso. Quanto mais atmosfera se atravessa, mais luz azul e roxa é espalhada e mais avermelhado aparece o Sol, o que acontece no nascente e no poente, em que a luz do Sol tem que atravessar a atmosfera tangencialmente.

Voce descarta Deus completamente da criaçao do universo ?

Certamente que sim. O Universo não foi criado. Surgiu espontaneamente. Deus não existe.

só para finalizar, sobre referenciais ainda, no caso de um referencial S que registre um movimento acelerado de um S' devido a uma certa força, no ponto de vista do S', o S está acelerando com força líquida igual a zero. Então, S é inercial, mesmo estando acelerando no ponto de vista de S'?

Sim. Como em S a aceleração é devida a uma força resultante não nula e seu valor está relacionado com essa força resultante pela relação R = ma, então S é inercial. Se houver aceleração mas ela não estiver relacionada com a força resultante pela relação R = ma, S não será inercial. A aceleração de S em relação a S', todavia ocorre sem que acha nenhuma força atuando sobre S, por parte de nenhuma interação física detectável. Então S' não é um referencial inercial.

''A vida vem em ondas como um mar, num indo e vindo infinito...''

Esta metáfora não é correta. Para começar a vida não é infinita e acaba algum dia. Depois, ondas são periódicas, isto é se considerarmos as cristas como os períodos de bonomia e os vales como os períodos de agruras, eles teriam que ser alternados sempre em tempos iguais. Isso não ocorre. Há gente para quem a vida é, toda ela, uma completa felicidade e outras em que seja uma completa amargura. E mesmo aquelas que, como acontece com a maioria, a vida tenha tempos felizes e infelizes, eles possuem duração completamente variada, não apresentando, em absoluto, a periodicidade das ondas.

A auto confiança se conquista na prática?

Sim, isso é possível. Como? Fazendo o que se tem que fazer sem auto confiança mesmo que, pouco a pouco, vendo-se que dá-se conta, ela vai sendo adquirida. Mas é preciso estar disposto a tentar e dar tudo errado e, mesmo assim, insistir outra vez, e outra vez e outra vez, ilimitadamente. Se desistir após vários fracassos, não se consegue. A teimosia é fundamental.

Envelhecer ainda é um tabu nesse país?

Só para alguns ignorantes. Quem tenha o mínimo de entendimento e consciência sabe, perfeitamente, que o envelhecimento é um processo normal da vida que todos os que não morrerem cedo passarão. Portanto o aceitam com tranquilidade e respeitam que tenha atingido esse estágio. Quem fica querendo continuar jovem em suas atitudes, estando velho, é uma pessoa totalmente imatura.

Um dia o senhro será apenas uma lembrança para nós aqui do ask, irá deixar essa vida e a unica pergunta que ecoara na mente de todos será ''existiu vida para ele após a morte?'' Os anos vão se arrastar até que nós descobriremos um a um pessoalmente.. todos estamos fadados ao mesmo fim.

Ninguém vai conseguir saber se existe algum tipo de vida não biológica após a cessação do funcionamento biológico de um organismo. Porque, como ele não está mais funcionando, não possui mais consciência. Em outras palavras, quem morre não existe mais, não tem mais consciência, não pensa, não tem sensações, não comunica nada. Então não há como saber se há algo para a pessoa já morta, porque não há mais pessoa.

Alguém pode ser muito bom em uma debate oral e ruim no debate por escrito?

Geralmente não. Pelo contrário, uma pessoa pode ser boa em debates por escrito e não o ser em debates orais. Quem consegue argumentar oralmente, também consegue por escrito, mas reciprocamente, nem sempre.

Ficar ansioso? Mas se nada o incomoda, onde está sua estoicidade? Esses remédios possuem uma série de efeitos colaterais, são tarja preta não por acaso.

O estoicismo é uma atitude racional. Emocionalmente situações estressantes provocam ansiedade de modo inconsciente. Como não ter dinheiro para pagar um empréstimo, ou não ter tempo para se desincumbir de uma tarefa ou algo do tipo. Os remédios possuem efeitos colaterais sim, mas os efeitos primários para o que eles são indicados compensam, por sua eficácia, os efeitos colaterais incômodos que possam provocar. Para isso é que existem remédios e eles funcionam porque nosso organismo é uma máquina química.

Ernesto, vários livros de apologética ficam afirmando que a ciência só surgiu por causa do cristianismo, que hitler era ateu, que stalin foi um ditador cruel por causa do ateísmo, etc. Você não acha que é escasso os livros de anti-apologética no BR p/ desmascarar o que estes livros espalham?

A ciência surgiu, exatamente, um milênio e meio depois dos gregos, que a faziam como filosofia, no Renascimento, a partir de pessoas que arrostaram o poder conservador da Igreja e se puseram a investigar o funcionamento do mundo à revelia do que dizia a Bíblia e os "Padres da Igreja". Se fosse deixar por conta do cristianismo, até hoje se consideraria que o mundo tivesse sido "criado" em seis dias e que o Sol giraria em torno da Terra e, mesmo, que a Terra fosse plana. Do mesmo modo que se consideraria que todas as espécies vivas teriam sido criadas diretamente por Deus. Hitler não era ateu, pelo contrário, era católico e Stálin foi cruel como foi não por ser ateu mas por ser um político totalitário que não admitia contestações a sua vontade. Não conheço livros em português que refutem isso, mas indico os seguintes:

O senhor toma atualmente toma algum remédio tarja preta?

Sim, Olcadil, no caso e estar ansioso ou tenso, já que isso é prejudicial a meu coração, que já sofre de insuficiência ejetora.

Na minha escola, um grupo de garotas estava querendo retirar as monótonas aulas de Educação Tecnológica e subistituir por aulas de redação, gostaria de saber sua opinião a respeito(esquecendo de fatos como leis jurídicas) PS: Eu amo computação, mas as aulas são realmente horrendas.

Educação Tecnológica é muito importante do mesmo modo que Redação. É bom ter tanto uma quanto a outra. Se as aulas são ruins é o caso de mudar o professor ou professora.

É a favor de psiquiatria? É a favor que se tome tarja preta?

Sim, sempre que for indicada, a psiquiatria é um recurso muito bom para a terapia dos distúrbios mentais. Da mesma forma que os medicamentos de tarja preta são muito bons para seu tratamento, bem como dos neurológicos, como epilepsia, por exemplo. Da mesma forma que esquizofrenia, depressão e, mesmo, tensão e ansiedade.

Como se deu a evolução da matemática, através dos tempos?

Isso é uma longa história que requer um livro para ser contada. Existem alguns bons, como o da Anne Rooney, da Tatiana Roque e do Carl Boyer.

Mas o que havia antes da explosão?

Nada. Conteúdo nenhum, nem espaço vazio, nem passagem de tempo. Isso tudo surgiu espontaneamente, sem ter de que provir. Só que não foi uma explosão, pois explosão é o lançamento de um conteúdo para um espaço circundante e o Big Bang foi o súbito início da expansão do espaço que continha e contém esse conteúdo, espaço esse que é uma função desse conteúdo e já surgiu expandindo-se. Com isso se deu, também, o surgimento do tempo, pois esse advém das alterações do estado do Universo e a expansão do espaço é uma alteração de estado. Note que, sendo como se pensa, o Universo infinito, ele já surgiu infinito, a expansão não significando um aumento de seu tamanho, o que é impossível, pois já é infinito, mas sim o aumento da separação entre suas localizações.

Professor, como você define a física teórica?

É a parte da Física que, justamente, com o uso da Matemática, baseando nos resultados experimentais e nas observações, bem como se valendo deles para verificar suas proposições, desenvolve modelos teóricos de explicação da realidade natural em seus aspectos mais fundamentais, básicos e profundos, ou seja, como se estruturam os componentes substancias do Universo, que são os campos, a matéria e a radiação e como esses constituintes interagem e alteram sua situação em razão dessas interações.

Ernesto, você possui alguma teoria sobre o que há além do Universo? Se a teoria do Big Bang diz que a explosão ainda continua, logo há um espaço vazio além do Universo, certo? E sempre houve esse espaço. Então sempre houve um Universo, até mesmo antes da explosão?

Não há espaço vazio nenhum além do Universo. Não há nada fora do Universo. Sequer espaço vazio. Tudo o que existe está dentro dele. A expansão do Universo não é uma expansão de seu conteúdo para um espaço vazio circundante e sim a expansão do próprio espaço, as coisas se afastando sem saírem do lugar, mas os próprios lugares é que ficam mas longe um dos outros. Se o espaço for infinito, como se pensa que seja, então o Universo não tem limite mesmo e o espaço se estende indefinidamente. Mas, mesmo que o Universo seja finito, ele não tem limite, não tem borda, não tem fronteira. Nesse caso, se se for andando sempre para frente, se chegará de volta de onde partiu, vindo por trás, após contornar o Universo que, então, seria curvado sobre si mesmo.

O senhor é um gênio! Mas de qualquer forma, acha importante programção ou lógica de programação importante para um físico?

Lógica é extremamente importante para qualquer cientista. E como, atualmente, tudo é feito em computador, mormente ciência, também é muito importante saber programar. Senão se fica preso a programas que já existem e não se inventa novos programas para resolver questões ainda não abordadas por ninguém.

Dizem que o senhor está pregando o anti cristo por estar possuído por demonios, o que acha dessa afirmação?

Quem diz que estou pregando o Anti-Cristo não acompanha nada do que escrevo. Não prego Anti-Cristo nenhum, pois, em meu entendimento, Cristo não foi nenhum Deus e essa história de Anti-Cristo, exarada no Apocalipse, é mera ficção. O que eu quero é a extinção de TODAS as religiões e que as pessoas não creiam em nenhum deus, bem como em anjos e demônios, em céu e inferno, simplesmente porque isso NÃO EXISTE. Que elas se libertem e vivam um vida feliz e virtuosa de forma inteiramente humanística, sem seguir nenhum líder, nenhuma doutrina, nem religiosa, nem ideológica, nem política, nem econômica. Claro que eu não estou possuído por nenhum demônio, já que eles não existem. Quem afirma isso está inteiramente equivocado ou é uma pessoa de má fé que quer me desacreditar com argumentos "ad hominem".

Qual a diferença de cálculo e análise? Acha interessante para um físico estudar análise?

Análise, como o nome diz, faz uma análise rigorosa de todas as proposições do cálculo, demonstrando-as de modo exato. Em geral, um físico não precisa disso.

Sobre professores não acertarem questões da Obmep, é de fato, um triste constatar da educação pública na rede estadual, meu ex-professor contou-me que o que ele sabe de matemática é o que aprendeu no Ensino Médio, e eu fiquei P-E-R-P-L-E-X-A, como assim? Como é que deixam isso acontecer? rsrs

Esse é um problema sério, advindo de que o salário de professor da Educação Básica é baixo e desestimula quem seja mais capaz a fazer licenciaturas e ser professor. Acaba acontecendo que os menos capazes tornam-se professores porque as licenciaturas possuem menor concorrência por vagas. Quem consegue entrar para engenharia, direito ou medicina, dificilmente vai preferir ser professor. Poucos preferem, mas são raros. Por isso que o curso de Física em que lecionei só tinha os ótimos e os péssimos alunos, mas não os médios nem os bons, nem os ruins. Todavia, em geral, os ótimos faziam mestrado e doutorado e se tornavam pesquisadores ou professores universitários. Então, para a Educação Básica, ficavam os péssimos. Uma lástima. Acho que um professor da Educação Básica deveria trabalhar em dedicação exclusiva, ganhando tanto quanto um professor universitário ou um oficial das forças armadas.

A ciencia nao eh apenas uma maneira humana de enxergar o mundo ?em outros lugares do universo ou ate entre nos mesmo poderia ser concebida inteiramente diferente da ciencia humana que conhecemos hoje ? A ciencia nao eh um fato absoluto ne verdade ?

Sim, a ciência é um construto humano e poderia ter se desenvolvido de forma diferente da que é, da mesma forma que, por outros seres inteligentes porventura existentes em outros planetas. O modo como ela modela a realidade é uma possibilidade dentre muitas, não sendo, de forma nenhuma, algo inescapável.

Professor, na sua opinião, um graduando de ciência da computação ou engenharia da computação teria alguma chance com a pós-graduação de física?

Sim, mas teria que fazer um nivelamento prévio das disciplinas do Bacharelado em Física não vistas em seus cursos, como métodos matemáticos da física, mecânica clássica e analítica, eletromagnetismo, física quântica e física estatística.

Tenho um aluno de 11 anos excelente em matemática, suas notas sempre são 10, mas uma coisa me intrigou ano passado: ele acertou apenas 2 questões da OBMEP. Existem crianças que mesmo apresentando, em sala de aula, excepcionais habilidades, não sejam superdotadas?

Sim. Tirar boas notas não indica superdotação. Ainda mais que, em geral, o nível do que se ensina nas escolas e se exige nas avaliações é muito mais baixo do que se cobra nas olimpíadas científicas e matemáticas. Por outro lado, superdotados podem, até, não tirar notas altas, porque não se interessam pelos conteúdos fáceis e se dedicam a outros mais difíceis e desafiantes. Para os alunos que pretendam participar de olimpíadas as escolas precisam oferecer aulas extras (e não poucas) para que os professores trabalhem com eles assuntos mais desafiantes ou os mesmos assuntos em nível mais elevado. O problema é que tem professor que, ele mesmo, não acerta muitas questões de olimpíadas.

O senhor vai na parada gay?

Não. Não gosto de aglomerados de pessoas. Não vou em nada desse tipo, seja sobre que assunto for. Como não vou a shows em que todo mundo fica assistindo de pé. Só em apresentações em salas de teatro em que todos ficam sentados nas poltronas. E em silêncio, ouvindo com atenção.

Ernesto, quero muito aprender sobre política, economia, história, filosofia, mas não sei como começar. Alguma dica?

Já disse o que fazer na resposta a sua outra pergunta. É só escolher os livros a respeito desses assuntos. Pesquise na internet e os compre, ou vá ler em bibliotecas.

Como adquirir mais conhecimento? Por onde começar?

Lendo, estudando, vendo vídeos. Comece lendo, pelo menos duas horas por dia, algum livro de divulgação de conhecimentos. Aos poucos aumente o tempo de leitura diário. Depois busque estudar, em livros técnicos mesmo, a respeito desse assunto. Enquanto isso vá já lendo livros sobre outro assunto e assim por diante. Para tal é preciso tirar o tempo de outras atividades para dedicá-lo a leituras. Como de ver televisão, de horas de sono (se dormir demais, pois o sono é necessário), de passeios, de barzinhos, de baladas e por aí vai.

https://ask.fm/wolfedler/answers/141396034333?utm_source=fb_share_own_ar#_=_ e o sol está em queda em relações ao planetas?

O Sol está em queda em relação ao centro de massa do Sistema Solar. Como a massa do Sol é extremamente maior do que a massa de todos os planetas reunidos, esse centre se encontra, praticamente, no centro do Sol.

Há como existir gradiente de pressão em um sistema fechado?

Claro. Ele sempre existe em razão da existência do campo gravitacional. Todo fluido, líquido ou gasoso, colocado em repouso em um recipiente fechado terá a pressão em sua parte inferior superior à pressão em sua parte superior.

Prof, você acha que dá para conciliar um Mestrado/Doutorado com um outro emprego ? Pergunto pq eu quero muito ser um cientista-professor mas não tenho quem me sustente e precisaria trabalhar. O preço da bolsa não sei quanto tá exatamente, mas acredito ser um valor meio insuficiente.

Sim, desde que você seja uma pessoa muito diligente, sem a menor preguiça e bem inteligente para não precisar estudar muito e aprender mesmo assim.

O senhor é Illuminati?

Nem illuminati, nem maçon, nem rosacruz, nem opus dei e nada desse tipo de coisa. Sou um livre pensador, desligado de qualquer concepção estabelecida, seja ideológica, seja religiosa, seja filosófica, seja psicológica, seja sociológica, seja política, seja econômica, seja física, seja matemática ou o que for.

Você acha que um dia algum planeta pode sair igual Plutão?

Os planetas de nosso sistema solar não vão se transformar em algo como Plutão. Ele é como é porque foi formado como é, em termos de massa e tamanho. O que pode mudar são as condições apresentadas em sua superfície, bem como em seus extratos profundos. Da mesma forma que os demais planetas.

O "vazio" que você se refere foi exatamente o que Wilczek mencionou em uma conversa com o Krauss (ambos físicos), sobre o vácuo. É o espaço que não contém nada que Newton imaginava que havia entre os corpos, onde a atração gravitacional se propagava instantaneamente (na gravitação newtoniana).

Não. Esse espaço não é vazio. Ele contém campo gravitacional, bem como radiação, proveniente das estrelas e galáxias e de fundo do Universo. Só não contém matéria. Trata-se pois, de um vácuo e não de um vazio. Vazio seria um espaço sem conteúdo nenhum: nem matéria, nem campos nem radiação. Isso não existe no Universo e nem fora dele.

então um carro se movendo aceleradamente em relação a uma árvore (referencial S), é a mesma coisa que a árvore se movendo aceleradamente em sentido oposto em relação ao carro (considerando em repouso em relação a si mesmo) (referencial S')? Então como saber nesse caso qual é o referencial inercial?

No referencial do carro ele não está acelerado mas há uma força resultante sobre ele. Por outro lado a árvore está acelerada e não há uma força resultante sobre ela. Portanto o referencial do carro não é inercial. No referencial da árvore o carro está acelerado, havendo uma força resultante sobre ele e a árvore não está acelerada, não havendo uma força resultante sobre ela. Portanto o referencial da árvore é inercial.

Os planetas não estão flutuando, estão em eterna queda no vazio.

Certamente que os planetas não flutuam, pois isso só acontece quando algo está imerso em um fluido. Os planetas orbitam, justamente porque estão em queda para o Sol. Só que não estão no vazio e sim no vácuo, que é um espaço sem matéria (ou quase sem), mas preenchido por campos, no caso o campo gravitacional do Sol, bem como pela radiação do Sol, de todas as estrelas e galáxias e de fundo do Univerdo. Como o espaço entre o núcleo e a eletrosfera de um átomo que não é vazio, mas vácuo, pois preenchido pelos campos elétrico e magnético do núcleo. Não existe espaço vazio no Universo (e nem fora dele).

Qual o significado da Lei de Coulomb ser igual a Lei da Gravitação Universal ? Se ouver outra propriedade atrativa-repelente, ele também teria a mesma estrutura das fórmulas supracitadas ?

A variação com o inverso do quadrado da distância para as interações gravitacionais e elétricas é uma decorrência do espaço ser euclideano e de que o fluxo do campo gravitacional e elétrico, uma vez emitido por suas fontes, ser uma entidade conservada. Assim, como a área de uma esfera com centro na massa ou na carga geradoras do campo é proporcional ao quadrado do raio, a intensidade do campo, que é o fluxo dividido pela área, se torna inversamente proporcional ao quadrado do raio. A proporcionalidade com o produto das massa e das cargas advém da definição do que seja a quantidade de massa gravitacional e carga elétrica. Já a proporção entre a massa gravitacional e a massa inercial, que permite o uso da mesma unidade de medida para ambas, é um fato da natureza que, inclusive, permite que se considere o campo gravitacional como um fenômeno inercial em espaço-tempos com curvatura.

Eu bebo e resolvo algumas problemas que nao resolveria se não bebesse, acha errado?

Acho. O álcool perturba o funcionamento do cérebro. Portanto as decisões tomadas sob seu efeito não são boas e bem ponderadas. É melhor resolver problemas estando sóbrio.

Prof Ernesto, se não for muito incômodo, você poderia fazer uma lista dos melhores livros de cálculo ? Estou começando estudar por conta própria pelo livro do George Thomas. O que achas ?

Foi o que eu estudei na graduação e acho bem bom. Também gostei dos livros do Maurer e do Kaplan & Lewis. Dos mais modernos, o Stewart também é bom. O melhor, contudo é o do Guidorizzi. Dos antigos, muito bom também, é o Courant. Citaria, ainda, o Piskunov e o Granville.

O que é ceticismo metodológico e ceticismo pirrônico?

O ceticismo pirrônico é a concepção apriorística de que não se pode atingir o conhecimento e a verdade. Já o ceticismo metodológico considera que se possa sim, mas que seja sempre preciso duvidar de que se o tenha atingido, para envidar esforços no sentido de, cada vez mais, validar esse conhecimento e conferir essa verdade.

Porquê a filosofia é importante? Qualquer linha de pensamento(ou compêndio) que seja fruto de bom senso e coerente é filosofia?Meu professor tenta,meio que impor(um pouco,rs)/sugerir valores éticos,e Morais,mas acho que é inútil,sendo que ele não pode mudar as pessoas de minha classe,e não mudam msm

A Filosofia é importantíssima, mais do que português, matemática, história, geografia, física, química, biologia, inglês e qualquer outra disciplina. Caia ou não nos exames de admissão aos cursos superiores. Porque a Filosofia é que faz uma análise da realidade, categorizando-a e permitindo a compreensão do mundo como ele é, bem como da vida em todos os seus aspectos, não apenas o biológico. O importante no estudo da Filosofia não é conhecer a sua história, como é feito na maioria dos cursos, mas, principalmente (mesmo que seja interessante conhecer a sua história), aprender a filosofar, isto é, a refletir sobre tudo, a analisar, a criticar, a cotejar, a contestar e não apenas aceitar tudo como é apresentado. Isso é fundamental para o encaminhamento da vida de modo que seja boa e significativa. Uma sabedoria intuitiva não é capaz, por si só, de proceder a esse trabalho. Estudando-se filosofia é possível entender porque é preciso ser ético, como encaminhar corretamente o raciocínio, a argumentação, como se classificam os diferentes conteúdos da realidade, como se validar ou não algum conhecimento, como se investigar a verdade e assim por diante. Filosofia teria que ser uma matéria com muito mais aulas por semana do que é dado normalmente (apenas uma). O professor precisa distinguir moral de ética e mostrar, de modo cabal, que a ética é algo inescapável, incontornável e não depende de qualquer circunstância, como a época, o lugar, o estrato social, a ideologia, a religião ou o que seja.

Por que na lei da gravitação universal é necessária a constante G? Para o que ela serve?

Para ajustar os valores experimentais nas unidades em que são medidos, já que as unidades de massa, distância e força foram definidas previamente de modo independente. Poder-se -ia criar um sistema de unidades para o qual a constante da gravitação fosse igual a um e, de fato, ele existe, onde tanto a constante da gravitação, quanto a de Planck e a velocidade da luz são iguais a um.

Por que provas de física que usam somente letras ao invés de números são ainda mais difíceis de entender?

Porque os alunos não estão acostumados, já que a maior parte dos professores de Física do nível médio não trabalham com letras, mesmo em exercícios e problemas que envolvam dados numéricos. Isso é muito ruim. Como professor de Física do Ensino Médio eu sempre resolvia qualquer exercício ou problema primeiramente com letras e, só no fim, substituía os valores numéricos. Isso desenvolve o raciocínio lógico e tem que ser feito desde o começo do estudo de Física.

Estás enganado...Mecânica clássica tem um conteúdo muito mais extenso que esses outros assusntos aí... Eu digo por que já estudei esses assuntos e, faltou ainda mecânica analítica. Conteúdo muito extenso e que exige muita interpretação do aluno

Discordo de você. Não só estudei mas fui professor tanto de Mecânica Analítica quanto de Mecânica Quântica (que não é a Física Quântica fenomenológica intermediária vista nas licenciaturas, mas uma disciplina de fim de curso do bacharelado). E a Teoria Quântica de Campos, que inclui a Eletrodinâmica e a Cromodinâmica Quânticas são mais difíceis ainda.

Serei um cientista melhor se eu for agnóstico ou ateu ?

Se você for um religioso cético, isto é, não dogmático, você poderá ser um bom cientista. Tem que estar aberto à possibilidade de sua religião estar errada.

Quais livros de física por nível de dificuldade alguém deve estudar se quiser adentrar em un mestrado? Só o Halliday já serve?

Não. O Halliday é um livro de Física Geral para o ciclo básico do Bacharelado em Física. No ciclo profissionalizante há disciplinas específicas com livros de nível intermediário, mais avançados do que o Halliday e similares, mas menos aprofundados do que os de pós-graduação, a respeito dos tópicos Mecânica, Mecânica dos Fluidos, Termologia, Eletromagnetismo, Ótica, Física Quântica, Relatividade, Física Estatística, Física da Matéria Condensada, Física Nuclear, Física de Partículas Elementares, bem como as Físicas Experimentais. Para fazer o mestrado em Física, antes tem-se que fazer o bacharelado, em que essas disciplinas são estudadas.

Existe algum ramo ou estudo da física que "estude" espíritos,paranormalidade e essas coisas? Se não,por quê não sanar de uma vez por todas essa dúvida de que exista espírito,paranormal,blabla?

ão existe nenhum ramo da Física dedicado a isso, exatamente por se tratar de algo inteiramente sem sentido, o que já está cabalmente estabelecido sem sombra de dúvida. Há várias obras explanando isso, como por exemplo:
God, The Failed Hypothesis - Victor J. Stenger
The Cambridge Companion to Atheism - Michael Martin
The Atheist Guide to Reality - Alex Rosenberg
Religion Explained - Pascal Boyer
God and the Folly of Faith - Victor J. Stenger
Why There is no God - Armin Navabi
The New Atheism - Victor J. Stenger
Deus, um Delírio - Richard Dawkins
Has Science Found God? - Victor J. Stenger
Quebrando o Encanto - Daniel C. Dennett
Faith vs. Fact - Jerry A. Coyne

Ernesto, o que é um tempo não-uniforme? E um tempo uniforme? Poderia explicar em detalhes?

O tempo pode ser próprio ou relativo. Próprio é o tempo medido por um relógio parado no referencial do observador. Relativo é o tempo medido por um relógio em movimento em relação ao observador. Chamando o primeiro de "s" e o segundo de "t", pode-se considerar a taxa de variação do segundo em relação ao primeiro, dt/ds, ou vice-versa, ds/dt. Se essa taxa for contante, diz-se que o tempo relativo corre uniformemente. Se ela for variável, ele corre de modo não uniforme. Isso vai acontece em função do movimento do relógio em relação ao observador ser uniforme ou não.

O Cálculo infinitesimal é o Cálculo Diferencial e Integral usual? Pergunto lhe isso pois encontrei em um sebo uma tradução de Michael Spivak denominada "Cálculo Diferencial e Integral Infinitesimal".

Cálculo diferencial e integral é o mesmo que cálculo infinitesimal. O título desse livro é redundante.

Como pode o universo se ter criado por si só?Há alguma chance de o universo ter um limite do tempo,algo como o "o fim do tempo",como se o tempo parasse(parece doido,eu sei)?O sol irá explodir,mas o universo irá provavelmente criar novamente a vida nesse espaço em que a terra ocupa?criando outro sol?

Da mesma forma que quem considera que o Universo tenha sido criado por Deus não sabe como é que ele teria feito isso, a concepção de que o Universo tenha surgido de nada (e não "do nada", o que é muito diferente), inteiramente por acaso, ainda não tem uma explicação de como isso teria se dado. Outra possibilidade é que o Universo tenha sempre existido. Note que ter tido um surgimento de nada por acaso não é o mesmo que ter-se criado a si mesmo. Outra observação é que, mesmo que se considere que o seu surgimento tenha sido uma criação por algum ente extrínseco a ele (se isso pudesse ser admitido que possa existir), isso teria se dado sem que seu conteúdo fosse proveniente da nada anterior, pois se assim o fora, esse algo de que o Universo teria sido feito já seria o Universo e, então, ter-se-ia que cogitar como é que tal conteúdo prévio teria surgido.

É possível que o universo seja um holograma e sem começo e nem fim,como um jogo maldito do acaso?(Vc deve ter uma ideia mais esclarecida)veja: https://www.scientificamerican.com/article/universe-really-is-a-holo/ https://phys.org/news/2015-02-big-quantum-equation-universe.html

Possível é, mas pouco provável. O mais provável é que o Universo seja materialmente real, isto é, o que percebemos como conteúdo (campo, matéria e radiação) é isso mesmo.

Professor, tem alguma mulher que o senhor admira? Seja escritora, pintora...

Várias: Hipatia de Alexandria, Madame de Chatelet, George Sand, Marie Curie, Lise Meitner, Ada Lovelace, Clara Schumann, Chiquinha Gonzaga, Hidelgard Bingen, Elis Regina, Frida Kahlo, Berthe Morisot, Emma Goldman e outras.

A frase ''Seja você a mudança que você quer ver no mundo'' está certa?

Sim. Se você pretende que o mundo seja de alguma forma, então é preciso que você também seja dessa forma, isto é, que suas concepções estejam de acordo com o que você queira que o mundo considere como certo, mesmo que seja discordante do que o mundo, agora, considere que o seja. Todavia, você pode conceder-se a viver de acordo com o mundo em alguns aspectos, diversamente de suas concepções ideais, para que consiga, simplesmente, viver. Por exemplo, eu acho que não deveria existir dinheiro no mundo. Então, de acordo com isso, faço muita coisa de graça. Mas não posso me abster totalmente de ter dinheiro, porque o mundo não me dá tudo de graça.

A ciencia humana nao tem a minima condiçao de supor grandes teorias sobre o universo,es tao limitada como o ser humano,sao coisas que apenas chegam ate onde a mente humana consegue conceber por um universo tao complexo e diferente.

Tem sim, claro que tem. Tanto que a ciência vem progredindo vertiginosamente nos últimos séculos. Isso pode permitir antever que, nos séculos e milênios vindouros, o progresso será maior ainda, sendo capaz de desvendar a maior parte do que ainda não foi descoberto e explicado. A complexidade do Universo é perfeitamente acessível à mente humana e, progressivamente, ela vai conseguindo destrinchar as mais complicadas ocorrências do mundo. Haja visto o grande avanço na Física das Partículas Elementares e da Cosmologia. Claro que o conhecimento disponível é incompleto e se tem muito mais de que não se sabe do que de que se sabe. Mas a ciência é extremamente jovem, podendo-se dizer que tem, apenas menos de meio milênio de existência. Esperemos passar alguns milênios. Por outro lado, a ciência tem a grande virtude de sempre ser provisória e jamais afirmar que já detém o conhecimento definitivo de nada. Nisso difere essencialmente das doutrinas religiosas que pretendem fornecer uma explicação definitiva sobre o mundo, inteiramente não fundamentada em nenhuma investigação com conclusões justificadas e comprovadas em termos dos conhecimentos disponíveis. O caráter dogmático das religiões as coloca em uma situação de completo descrédito para prover qualquer explicação, seja a respeito do que for.

Por quê o Brasil ,na maioria das vezes, esta em um posicionamento baixo no Ranking mundial das melhores universidades ?

Porque não existe uma política governamental agressiva em relação à qualidade das universidade, públicas ou privadas, que invista pesadamente nelas e, por exemplo, contrate, com salário diferenciado para mais, pesquisadores de alto nível internacional para desenvolverem os projetos de pesquisa e os programas de ensino. Porque, também, o nível de exigência para aprovação dos alunos é muito frouxo, de modo geral.

voce disse q um referencial só inercial SE, em relação a ele, algo não estiver experimentando aceleração. Mas imagine um carro e árvore. Se a árvore experimenta aceleração em relação ao carro, o carro não é inercial, pois em relação a árvore, o carro se move aceleradamente? pode explicar ?

Não foi isso que eu disse. O que eu disse é que um referencial é inercial se, em relação a ele, um corpo QUE ESTEJA SOB A AÇÃO DE UMA FORÇA RESULTANTE NULA POR PARTE DE TODAS AS INTERAÇÕES QUE ATUEM SOBRE ELE, não possuir aceleração e, reciprocamente se, em relação a ele, um corpo que não apresente aceleração, ESTEJA SUBMETIDO A UMA FORÇA RESULTANTE NULA POR PARTE DE TODAS AS INTERAÇÕES QUE ATUEM SOBRE ELE. No caso do referencial ligado ao carro, se a árvore se mostrar acelerada nele, sem que esteja sujeita a uma força resultante não nula, então esse referencial não será inercial, ou seja, ele terá uma aceleração em relação a algum referencial inercial.

Curso o 2 ano do Ensino Médio, porém encontro-me muito indeciso quanto a minha área de interesse tens alguma sugestão ?

Sim. Seja Físico ou Filósofo. São as ocupações mais interessantes e fascinantes que se possa ter. Ou, então, vá ser repórter da National Geographic Magazine. Também é interessante ser piloto de provas de aeronaves ou astronauta. Ou, ainda, ser escritor, compositor de música clássica, pintor (de quadros, não de paredes), arquiteto, artista de cinema ou de novelas da televisão, dramaturgo. Tudo que for bem criativo e incomum.

Eu seria burro por tirar zero em uma prova? Minha autoestima está destruída! Certo que eu não estudava nem prestava atenção nas aulas... será que souburro mesmo? É normal tirar zero? Se eu fosso inteligente, mesmo sem estudar, nem prstar atenção nas aulas, tiraria total

Um zero em uma prova não é sinal de burrice. Mas notas ruins sistemáticas é sim. Ou então de grande falta de empenho nos estudos. Quanto a tirar notas altas sem estudar é possível, mas, nesse caso, tem-se que prestar muita atenção nas aulas para que, só com isso, sendo-se bem inteligente, se aprenda. As notas são uma consequência de se ter conhecimento e habilidade em usá-lo.

Me diga por quê eu deveria começar a estudar hoje? Me dê um motivo para isso...

A maior razão para estudar é a sede de conhecimento. O resto é consequência. Você só deve começar a estudar se for isso que você gostar de fazer. Se não, não comece. Não é preciso estudar para ser feliz e nem para contribuir para o bem do mundo. Mas, estudando, pode-se ser muito feliz e dar uma grande contribuição para o bem do mundo.

Vamos supor que esse ano um pesquisador brasileiro x conquistasse um premio nobel qualquer. E ele fosse condecorado e convidado para sessões solenes pela sua conquista pelo poder executivo. Ele recusa por questões políticas, seria uma atitude anti-ética?

De modo nenhum. Pelo contrário, seria extremamente ética ao manifestar taxativamente sua discordância com a situação política atual, criando um verdadeiro escândalo de repercussão internacional, o que seria muito bom.

Em que se baseia para afirmar que a Igreja Católica não aceita descobertas científicas. Poderia citar alguns exemplos?

A Igreja Católica, mesmo aceitando as Teorias do Big Bang e da Evolução, não aceita que o surgimento do Universo e da Vida possam ter se dado por acaso. Nem aceita que todo o psiquismo seja o resultado da anatomia e fisiologia do cérebro apenas. Além de aceitar ocorrências, como milagres e atendimento de orações como verdadeiras.

Você estava certo mesmo, física, principalmente mecânica clássica e termodinâmica são as matérias mais difíceia que existem

Mecânica Clássica e Termodinâmica não são muito difíceis. Eletromagnetismo, Mecânica Quântica, Relatividade Geral e Teoria Quântica de Campos são mais difíceis.

o que é "roskoff"?

https://ciberduvidas.iscte-iul.pt/consultorio/perguntas/roskoff--roscofe/14254
https://www.dicio.com.br/roscofe/
https://www.priberam.pt/dlpo/roscofe

Tens mais pena de seres humanos ou de animais domésticos?(Ao presenciar uma agressão, por exemplo).Eu tenho mais pena de animais, seria isso uma falha de caráter?

Também tenho mais pena de animais e crianças do que de humanos adultos. Porque eles são mais indefesos em relação à perfídia de algumas pessoas. Todavia, no caso de ter que escolher, por exemplo, entre salvar uma pessoa humana ou um animal que estejam em luta, eu salvaria a pessoa.

tenho algumas perguntas: fazer mestrado em física em teoria de campos e relatividade geral é para qualquer um? quanto de qi você acha que é necessário para se graduar em física sem apresentar muita dificuldade? Você disse que física é para pessoas inteligentes.

Qualquer estudo feito em nível superior não é para qualquer um. Isso inclui Direito, Administração, Pedagogia, Filosofia, Biologia, Medicina, Engenharia ou o que for. A não ser que a pessoa faça um curso bem roskoff. Todavia, Física e Matemática, em verdade, exigem um nível maior de inteligência no aspecto lógico e uma intuição a respeito do comportamento da natureza. Do mesmo modo que uma facilidade de expressar ideias. O mesmo acontece com a Filosofia. Estou me referindo a quem vá levar seus estudos de Física e Matemática (ou Filosofia) até um nível de pós-doutorado, mesmo que, formalmente, não faça esses cursos. Quanto a encontrar alguma dificuldade, mesmo quem seja bem inteligente vai encontrar nesses campos. É uma característica intrínseca deles. Requerem muita dedicação ao estudo.

A matemática não foi inventada. A matemática é a priori. A matemática existe independende de nós. 2 + 2 serão 4 independe do ser humano. Em qualquer mundo possível (Leibniz) a matemática se mantera. O que nós inventamos/criamos foi a linguagem para expressar a matemática.

Acontece que a matemática é, exatamente, a expressão dessas relações apriorísticas entre números e figuras em uma codificação por meio de uma linguagem de símbolos com regras de manipulação que, para que sirvam para algo, devem estar de acordo com o que acontece na realidade. Isso que acontece na realidade não é a matemática e sim o modo como a realidade é, que não depende de algum ser provido de inteligência que a decodifique. Só que essa decodificação é uma invenção sim. A matemática não descobriu essa linguagem, ela a inventou.

Nos concursos da ABIN(agência nacional de inteligência) valoriza-se muito que as pessoas conheçam línguas como árabe e outras das quais não me recordo. E a prova da língua escolhida é realmente difícil, um linguajar muito técnico.

É porque eles se voltam para o problema do terrorismo e da possibilidade de alguma revolução dita "comunista", então precisam que seus agentes entendam árabe, chinês, russo e por aí vai. Quem não está voltado para isso, mas se liga mais na contribuição literária, cultural, filosófica e científica produzida em algum idioma, vai preferir saber francês, alemão, inglês, italiano, russo, espanhol do que essas. Bem como grego e latim.

Hoje é o 89º aniversário de Che Guevara. O que vc pretende fazer para homenageá-lo?

Acho que Guevara foi um facínora que não merece homenagem nenhuma. Como Fidel Castro. Não foram comunistas. Foram déspotas e assassinos. O dito "comunismo" soviético, cubano, chinês e outros, em verdade é uma grossa traição aos ideais comunistas por parte de Lênin e seus seguidores, que só substituíram a opressão da aristocracia e da burguesia pela opressão da máquina do partido dito impropriamente "comunista". O bolchevismo é tenebroso.

Ernesto, sendo um ateu esclarecido como você é, não concordas que se Tomás de Aquino fosse competente na arte de pensar, ele deveria ter compreendido com facilidade que sua teologia e religião eram totalmente equivocadas?

Pois é. Tomás de Aquino era inteligente, mas não foi um verdadeiro filósofo. O que faltou nele foi o ceticismo e o livre-pensamento. Ele tinha uma grande habilidade argumentativa e era muito didático (em verdade só li dele vários trechos de sua "Suma contra os Gentios" e não a "Suma Teológica"). Mas subordinou a filosofia à teologia e buscava sempre (até que com grande talento), distorcer os argumentos filosóficos para que não discordassem dos teológicos. Diferente dele foram Duns Scoto e Guilherme de Ockham, que eram filósofos mesmo mas que aceitavam as propostas religiosas do cristianismo por fé, mostrando que a razão não era capaz de confirmá-las.

Por quê acredita que a religião Católica seja pior que outras (o Islamismo,por exemplo)?

Não só o catolicismo, mas o cristianismo em geral. Porque ele se fundamente em uma crueldade ímpar de seu deus, que é a redenção de Jesus Cristo. Para que esse deus aceitasse a humanidade no paraíso, ele exigiu um sacrifício expiatório imenso que foi a morte de seu próprio filho em sofrimentos atrozes. Ora, se é um deus de bondade, porque simplesmente não perdoou? A redenção da humanidade por meio da imolação de Cristo na cruz é uma crueldade sem par do deus cristão. Não quero ser amigo de um sujeito assim. Além do mais, isso só se explica na hipótese de que a humanidade tenha sido criada nas pessoas de Adão e Eva, que cometeram o pecado original. Como isso é, simplesmente, uma lenda, então a redenção não tem significado nenhum. O islamismo também é uma religião enganadora, só de supor que exista um deus. Mas sua teologia não incorpora uma crueldade dessas. Por outro lado, sua ética é mais cruel do que a cristã. Além de subordinar toda a vida social à vida religiosa. O judaísmo e o cristianismo antigos também eram assim. Na Idade Média e na Moderna os reis reinavam por direito considerado divino. Quanta bobeira. O direito deles era um direito advindo da tomada do poder, geralmente com o uso da força, por algum ancestral do rei atual.

A matemática foi inventada ou descoberta? Discorra.

Foi inventada. Todavia a matemática que permanece usada é aquela que está de acordo com o funcionamento da natureza e da sociedade. Outras formas existem e são estudadas, mesmo que não tenham utilidade nenhuma. É interessante ler este artigo:
https://www.dartmouth.edu/~matc/MathDrama/reading/Wigner.html

Boa noite, professor; primeira vez perguntando aqui. Acredita no livre arbítrio? Qual é o seu conceito ou concepção do mesmo? E a existência do livre arbítrio não seria mutualmente exclusiva a um universo determinista? Agradeço desde já.

Não é uma questão de acreditar e sim de constatar. De fato, existe. E isso é uma boa prova de que o Universo não é determinista, pois, se fosse, não haveria livre-arbítrio.

Ernesto, a coleção anarcobolsos da editora Hedra é suficiente para compreender o anarquismo? Existem outros livros contemporâneos sobre o tema?

Não conhecia essa coleção, mas, examinando-a, achei bem boa. Realmente os livros dão uma panorâmica do anarquismo. Outro que recomendo muito é o livro; "História das Idéias e Movimentos Anarquistas", de George Woodcock (em dois volumes).

o que a relatividade geral diz respeito a referenciais que estejam com uma aceleração não constante, pois para ser não inercial, precisa ter aceleração constante?

Para não ser inercial um referencial tem que ter aceleração com relação a outro que seja inercial. Mas a aceleração não precisa ser constante. E um referencial é inercial quando, em relação a ele, um corpo que não possua aceleração, está sujeito a uma força resultante nula por parte de tudo que interaja com ele e, reciprocamente, se a resultante das forças devido a todas as interações existentes for nula, ele não possua aceleração nesse referencial.

Acho um absurdo ,uma censura eu perder contas por falar com o senhor, eles me vêem como alguém que ofende o senhor e que viola as regras do site.

Eu não me sinto ofendido com pergunta nenhuma, mesmo que discorde da opinião de quem as faça. Há tempos havia uma pessoa no Ask, chamada Hans, que me bombardeava de críticas por eu não ser nazista como ele. Sempre o respondia com argumentos serenos até que ele parou. Não acho que ninguém deva denunciar ninguém em razão de perguntas que me façam, que a pessoa não concorde se, eu mesmo, não me incomodo em respondê-las e nem as acho ofensivas.

professor, então quando há um movimento relativo de um elétron que esteja submetido a um campo elétrico, então o campo magnético surge? então o campo magnético é uma abstração?

O campo magnético é um efeito relativístico que aparece quando um observador está em movimento em relação a uma partícula carregada ou a um campo elétrico. Daí a entidade ser o "campo eletromagnético", que se manifesta parte como elétrico, parte como magnético, essas partes dependendo do movimento do observador.

mas então um elétron pode ser maior que um átomo, considerando suas propriedades ondulatórias? E quanto mais rápido um elétron estiver, essa onda vai se estender por um "comprimento maior"? E o que isso tem a ver com a incerteza de Heisenberg?

Quando ligado aos núcleos formando átomos, os elétrons têm o tamanho do átomo todo. Quanto isolados não. Sua extensão espacial é bem menor (mais de dez mil vezes menor). O princípio da incerteza tem a ver com a medição da posição e do momentum. De fato, essa incerteza está relacionada com o fato de que o campo associado à partícula se espraia por um região e não se localiza em um ponto.

no Halliday, diz que fora as correntes elétricas, há o campo magnético intrínseco de elétrons e outras partículas. De onde esse campo advém?

Do fato dessas partículas possuírem carga, mesmo que a carga total seja nula, como no caso do nêutron e de, também, possuírem momento angular intrínseco (spin). Classicamente é como se fossem esferas carregadas girando. No caso do nêutron a questão é que ele é composto de três quarks e eles possuem carga e spin, portanto momento de dipolo magnético (como se fossem ímãs), sendo que o o momento de dipolo magnético total não é nulo, mesmo que a carga total o seja.

Análise real é muito complicado pra mim, que só sei cálculo, é muito rigoroso. Métodos matemáticos envolvem conteúdos que não verei no meu curso.

Se o seu curso é de engenharia, o estudo de métodos matemáticos da física é muito proveitoso. Além disso, nunca é demais saber além do que seja preciso.

o comprimento de onda de um quanta é, em geral, o lugar que esse quanta deve estar no espaço? e fora dessa região "ondulatória", pode ser que se encontre também, mas sendo muito improvável? li que os elétrons do universo, podem estar em qualquer lugar, mas como menos probabilidade dependendo.

o comprimento de onda de um quanta é, em geral, o lugar que esse quanta deve estar no espaço? e fora dessa região "ondulatória", pode ser que se encontre também, mas sendo muito improvável? li que os elétrons do universo, podem estar em qualquer lugar, mas como menos probabilidade dependendo.

Você pode me indicar demonstrações de teoremas, integrais extremamente difíceis?

Ao invés de estudar por um livro de Cálculo, pegue um livro de análise real que os teoremas são mais difíceis. Ou um livro de métodos matemáticos da física.

Por que tens muito interesse em linguística?(O que vê de interessante)

Primeiro que eu acho fascinante a forma como a humanidade inventou a língua como um meio de comunicação tão poderoso que é. Então gosto de entender como uma língua se estrutura, que diferenças elas guardam entre si e outros aspectos estáticos. Além disso, acho mais fascinante, ainda, o estudo da evolução das línguas, umas a partir das outras. Não por isso ter aplicação nenhuma mas, apenas, para me maravilhar com esse conhecimento.

Professor, se o gráfico de uma função possui assíntota horizontal, então esse gráfico não possui assíntota oblíqua, correto?

Pode ter sim. Uma horizontal quando as abcissas tendem para infinito positivo e uma oblíqua quando as abcissas tendem para infinito negativo. Ou vice-versa.

Acha que árabe vale a pena aprender?

Acho que não. Da mesma forma que japonês, chinês, hebraico. São idiomas muito difíceis, de estrutura muito diferente dos idiomas indo-europeus e não possuem uma literatura de tanta monta que compense o seu estudo.

'' um deus, verdadeiramente único é preferível do que a de um deus de tripla personalidade.'' Por que seria melhor?

Porque, se existir algum deus com as características que se atribuem a essa entidade, ele, necessariamente, tem que ser único. Ora, a concepção de um deus com três pessoas, como o judaísmo e o cristianismo o fazem é um politeísmo disfarçado, sendo, assim, inconsistente com o que se entende por um deus. O próprio Tomás de Aquino, mostrou a necessidade da unicidade do deus. Todavia tergiversou ao considerar sua tripla personalidade. Só que essa é uma discussão estéril, pois, afinal de contas, não existe deus nenhum. Mas a concepção islâmica é mais consistente. Ela se aproxima mais da concepção deísta, que é mais consistente ainda, apesar de também ser falsa.

domingo, 4 de novembro de 2018

Professor, o senhor é rigoroso com suas medicações?

Um tanto, mas nem tanto. Às vezes esqueço de tomar. Um tanto, mas nem tanto. Às vezes esqueço de tomar. Mas na maioria das vezes tomo.
na maioria das vezes tomo.

Professor, o senhor é rigoroso com suas medicações?

Um tanto, mas nem tanto. Às vezes esqueço de tomar. Um tanto, mas nem tanto. Às vezes esqueço de tomar. Mas na maioria das vezes tomo.
na maioria das vezes tomo.

Fiz bem em cortar uma relação sem reciprocidade com uma pessoa que conheci na internet?

Se isso te deixou melhor, sim. Se isso te deixou passado, não.

Então, caridade é apenas uma porcentagem miníma do que traz prazer?

No meu caso, não. É uma parte menor, mas não uma fração mínima. Outras pessoas pode ser que seja e, outras ainda, que seja uma fração maior. O que não significa que eu não goste de fazer caridade ou que isso não me gratifique. Mas isso não é o que me dá mais prazer na vida. Não se pode extremizar as opções. Têm que ser devidamente ponderadas.

"cavalos sao muito uteis ao homem". isso no inges ficaria "horses are very ufesul to man" ou "horses are very useful to the man"?

Escolho a primeira opção. Mas não sou professor de inglês.

Mas por quê o 3 apareceu multiplicando? Como se chegou a isso? Visto que não estávamos derivando e sim integrando? Só quero saber se usa- se MMC, no primeiro membro, seria isso? Visto que o 3 apareceu dividindo todo membro da equação e pela conservação do valor, ele aparece no numerador também.

A integral ∫π(r² - x²)dx = πr²∫dx - π∫x²dx = [πr²x - πx³/3] (x = -r a x = r) = πr³ - πr³/3 + πr³ - πr³/3 = 2πr³ - 2πr³/3 = 4πr³/3.

Por que sânscrito seria válido aprender? Além do mais, acha que árabe valeria a pena?

Porque a literatura védica é muito interessante, como o Mahabharata e os Upanishads. Além do mais o Sânscrito é o idioma mais próximo do proto-indo-europeu, do qual derivam os idiomas latinos, germânicos, eslavos, grego, persa. Tenho muito interesse em linguística.

Por que quando se aplica a integral a fórmula V=∫r -r π y^2 dx Substituindo em y=sqrt r^2- x^2 V=π [r^2 x - x^3/3] r -r V=π [ (3.r^3- r^3/3) - (-3.r^3 + r^3/3) Minha dúvida é por quê o 3 aparece multiplicando como se fosse derivar? Então subtraria uma unidade do expoente não?

O elemento de volume a ser integrado é um disco cilíndrico, de raio da base y e altura dx, cujo valor é dV = πy²dx. Como r² = x² + y², sendo r fixo e x a variável de integração, é preciso expressar y em função de x, o que dá y² = r² - x². Então o elemento de volume fica dV = π(r² - x²)dx. Portanto o volume será a integral disso entre os limites x = -r e x = r.

Russo, Francês ou Alemão, qual você acha que vale mais a pena?

Francês. Trata-se do idioma mais cultural dos três. Todavia os outros também são muito interessantes, bem como o italiano, além do inglês e do espanhol. Depois desse seis, além do português nativo nosso, acho válido se aprender grego, latim e sânscrito.

Ernesto, em adultos o hábito de leitura altera a cognição?

Sim, para melhor. Quanto mais se ler, especialmente ler algo que requeira reflexão para ser entendido e consulta ao dicionário para ver o significado das palavras, mais se desenvolvem conexões sinápticas no cérebro, o que significa que se aumenta a inteligência, justamente no aspecto da cognição e do encaminhamento de raciocínios. Leituras amenas não desenvolvem tanto isso, mas também ajudam. Muito mais do que assistir vídeos, porque a leitura requer que se imagine as cenas descritas, que não são apresentadas prontas, como nos filmes e vídeos. A regra geral para desenvolver a inteligência consiste, justamente, em complicar. Tudo o que simplifica, emburrece. Portanto não é bom que professores simplifiquem tudo. Têm que começar de forma mais simples e acessível, mas não ficar nesse patamar e sim ir, gradativamente, elevando a complicação, para que o estudante desenvolva a inteligência. Porque um dos objetivos da educação, além de transmitir conhecimentos e desenvolver habilidades, dentre outros, é aprimorar a inteligência. Mas o mais importante é formar o caráter. Veja isto:

Não entendi por quê na dedução do volume da esfera por integrais fica (y=sqrt r^2-x^2)

Dado um ponto de coordenadas "x" e "y", sobre uma esfera de raio "r", a ordenada, a abcissa e o raio formam um triângulo retângulo de catetos "x" e "y" e hipotenusa "r". Então, pelo teorema de Pitágoras, r² = x² + y². Logo...

O que acha dessas dicas do Feynman para aprender :http://www.bbc.com/portuguese/geral-36750825

Ótimas. Em suma o que ele diz é o que eu propugno: a melhor forma de aprender é ensinando. Mas tem que ensinar de modo que os outros aprendam.

Quais aspectos positivos há no islã?

A concepção de um deus, verdadeiramente único é preferível do que a de um deus de tripla personalidade. A rejeição do papel de Jesus como redentor. O preceito de fazer caridade.

Tem que se levar a vida com alegria ou seriedade? O que precisa predominar no dia a dia?

Ambas, sempre. Elas não se excluem, pelo contrário, se complementam.

Como fazer para o proletário ter o controle dos meios de produção?

Para começar, o que é preciso não é dar o controle dos meios de produção e dos resultados da produção ao proletariado. É acabar com o proletariado, ou seja, transformar toda a população em burgueses. Isto é, eliminar o emprego e o trabalho assalariado e fazer com que todo trabalhador seja autônomo ou sócio do empreendimento em que trabalha. Assim não se tem nenhuma distinção entre patrão e empregado. Todos são patrões, todos são trabalhadores, todos são capitalistas. Isso se alcança gradualmente por meio de uma legislação que vá distribuindo o capital das empresas entre seus trabalhadores até que ninguém seja empregado. Numa etapa posterior (mas isso leva séculos) se abole totalmente o dinheiro e a propriedade de modo natural, sem revolução e sem leis. Simplesmente porque não há mais necessidade de que exista. Tudo passa ser compartilhado. Inclusive mulheres e maridos.

Ensinar sobre ateísmo não faz as pessoas pensarem em ajudar os outros automaticamente, use mais seu tempo livre para ensinar algo relevante, em vez de querer mudar opiniões religiosas.

De modo nenhum. Difundir o ateísmo é algo extremamente relevante. Muito mesmo. Mas não pode ser dissociado de outros ensinamentos, como a ética, por exemplo. Bem como o espancamento da ignorância científica e cultural. A ética ateísta é muito mais válida do que a religiosa, porque concita à prática do bem sem se reportar a nenhum castigo e a nenhuma recompensa, mas tão somente ao valor intrínseco do bem. Assim é que as pessoas têm que agir. Sob a égide da ética, também se abriga o comunismo e a anarquia, que são os mais perfeitos e éticos sistemas, respectivamente, econômico e político. De modo que, pregar o ateísmo, o comunismo e a anarquia são as mais éticas ações que se pode fazer para promover o bem do mundo. Isso é que é de extrema relevância. Mudar as opiniões religiosas das pessoas, para que elas deixem suas crenças numa realidade sobrenatural é a máxima caridade que se pode fazer com elas. Em segundo lugar vem o fomento da prosperidade econômica (que não significa prosperidade financeira, numa economia não monetária e nem de escambo, como deve ser, idealmente, a economia). Nisso se encaixa a elevação do nível de saúde da população. Mas o mais importante é a educação. Educação para o ateísmo, para a anarquia, para o comunismo e educação científica, artística, cultural e, principalmente, ética.

Como se conceitua temperatura para uma única partícula? Além dessa dificuldade, é costumeiro dizer que a radiação cósmica de fundo tem temperatura média de 2,75 K +-. Como é feito para ondas/radiação? Até então, só vi referências à energia cinética translacional de um aglomerado de partículas.

Não existe temperatura de uma única partícula. Só de sistemas de muitas partículas. Temperatura é um conceito estatístico. No caso da radiação, pode-se considerá-la como um gás de fótons e se obter a temperatura a partir da energia média dos fótons. A dedução é um tanto complicada. Veja isto:

Para você, qual o foco de estudo mais interessante da Física?

Relatividade Geral e Cosmologia, bem como Física de Partículas Elementares e Campos.

Reclamas da Europa por estar impregnada de cristianismo? Quero ver quando a Europa se tornar Eurabia e estiver impregnada de islamismo, ai sim quero vê-lo reclamar de cristianismo.

O islamismo é tão ruim quanto o cristianismo. O que eu queria é que a Europa estivesse impregnada de ateísmo, livre de qualquer religião. Se fosse para haver uma, eu preferiria o budismo. Todavia há aspectos positivos tanto no cristianismo quanto no islamismo. Mas eles não compensam seus aspectos negativos, que são só deles, enquanto os positivos podem ser levados adiante sem as religiões. O pior aspecto das religiões é o fato delas serem enganações, isto é, mentiras. Sejam quais forem.

O senhor tem um universo interno?

Claro. Todo mundo tem.

Alcançando e estendendo a mão e ajudar as pessoas vai lhe trazer mais satisfação do que qualquer outra coisa que já fez?

Não. O que me traz mais satisfação é amar, pensar, ler, estudar, criar (escrever, pintar, compor, poetar). Todavia a caridade é algo que é preciso ser feito, mesmo que não propicie tanto prazer pessoal, em razão do objetivo de consertar o mundo, uma vez que ele ainda não é perfeito e há muita gente necessitada. Então esse esforço acaba, também, se revelando gratificante, mesmo que não prazeroso. Mas é feito com amor.

Por que o senhor não quer ser africano?

Porque eu me identifico com o modo de ser daqueles países que eu mencionei (Noruega, Austrália, Canadá). Se houvesse algum país na África daquele jeito, eu gostaria de ser morador dele. Não é porque são brancos e sim porque são como são, em termos de disciplina, de ordem, de progresso, de cultura refinada, de honestidade, de diligência, enfim, de civilização. Os países africanos são muito avacalhados para o meu gosto. Como, em geral, os árabes. O Japão também é um país assim e lá, mesmo não tendo uma civilização européia, eu gostaria de viver. Mas eu me identifico mais com a civilização européia. Pena que ela seja muito impregnada de cristianismo.Nesse sentido eu preferiria que fosse impregnada de budismo ou, preferencialmente, de ateísmo. Acho que se o Império Romano tivesse continuado pagão seria melhor.

O senhor fumaria maconha para aliviar alguma dor se essa fosse a sua unica opção?

Fumar não, pois detesto. Mas ingeriria o princípio ativo da maconha por outro meio, como ingestão ou injeção.

Por quê os alunos bocejam tanto em aulas monótonas ou chatas?

Ora, porque elas são monótonas e chatas. Aulas têm que ser participativas, polêmicas, interativas, com muita discussão, vibrantes, cativantes, gostosas. Quem não conseguir assim fazer a sua aula não é bom professor ou professora. Mas não precisam ser caricatas e gaiatas. E têm que por os estudantes para trabalhar e pensar. Nada de assistir passivamente a exposições verbais ou a projeções de slides ou filmes. Isso é válido num contexto interativo.

O senhor acha que está ajudando a mudar o mundo respondendo perguntas sobre física e matematica? Perguntas essas que disse sentir mais vontade de responder do que outras.

Claro que sim. Diminuir a ignorância é aprimorar o mundo. E uma das ignorâncias mais disseminadas é a científica. Quem tem mais conhecimento científico (e isso inclui a matemática) tem mais facilidade de aceitar os argumentos contra a existência de deuses, por exemplo, fator importantíssimo para o progresso civilizatório da humanidade. Sem mencionar que o conhecimento científico também contribui para a melhoria das condições tecnológicas da humanidade, o que possibilita a redução da pobreza e das doenças, por exemplo. O que não se pode é ter conhecimento científico apenas, sem estar associado ao conhecimento das humanidades e das artes, bem como da filosofia. Especialmente para o desenvolvimento das noções éticas e sociais.

Mesmo os criminosos possuem direito a um advogado; os que parecem culpados podem ser inocentes.Há os que confessaram sob pressão. A pena possui graduação, pois podem ter vários crimes e se ser culpado por um e não por outros.Podem ter várias pessoas envolvidas, aumentando a complexidade.

Sim, mas se eu fosse um advogado e descobrisse que meu cliente fosse culpado mesmo, eu não o defenderia em hipótese nenhuma. Não vejo como se pode defender alguém que seja culpado. Isso, para mim, é um absurdo.

Professor, como um profissional de direito pode dar sua contribuição para um sociedade mais justa, solidária e fraterna? Como seria um perfil de um profissional de direito ideal para você? Como seria seu papel na luta de uma sociedade melhor?

Defendendo os inocentes injustiçados e processando os malfeitores, opressores e criminosos em geral. Recusando-se sempre a defender quem não tenha razão, mesmo que, com isso, deixe de ganhar dinheiro. Disponibilizando-se a trabalhar de graça para quem não tenha condições de pagar e precise dos serviços de um advogado por ter sido prejudicado, sendo inocente. Pautando seu trabalho pela mais completa lisura, honestidade e ética. Não se furtando a defender o bem, mesmo com prejuízo ou com risco, até de morte.

Se tivesses de escolher uma nacionalidade diferente, qual escolherias?

Talvez norueguesa. Ou australiana. Ou canadense.

Deveria ser prazeroso, deveria se sentir bem ao fazer, se sentir orgulhoso, util, sentir que está vivendo pelo certo!

Ora, fazer o bem é prazeroso, propicia bem estar, sentimento de utilidade e sensação de se estar vivendo do modo certo. Mas não é para se sentir assim que se tem que fazer o bem e sim pelo simples valor intrínseco do bem, sem objetivar nenhuma recompensa e sem o fazer por temer nenhum castigo se não o fizer. Porque esse castigo não existe e nem sempre se é recompensado. Nem por isso não se há de fazer o bem.

Felicidade em primeiro lugar?

Não. Inclusive porque não se obtém a felicidade quando se a procura. Ela vem como brinde ao se buscar fazer o bem. Esqueça a felicidade e se foque no bem que se a alcança.

Ser perfeccionista, querer entender tudo em mínimos detalhes, com várias deduções e, consequentemente, levando mais tempo para se aprender, implica em ser burro? Já que despenderá mais tempo?

Da modo nenhum. Pelo contrario. Em geral por não se ser burro é que se quer saber tudo, tintim por tintim. A questão do menor tempo de aprendizado significar maior inteligência se aplica no caso de se estar aprendendo o mesmo, no mesmo nível de pormenores. Se se quiser aprender mais detalhadamente, mesmo sendo mais inteligente, certamente se demandará maior uso de tempo.

Acho que precisa existir o meio termo, não se pode viver apenas para ajudar os outros, as pessoas tem aspectos pessoais que precisam evoluir e precisam de um tempo para elas mesmas, o desgaste em esquecer de si pelos outros é um caminho perigoso, precisa haver um controle, não acha?

Certamente que sim. E nem precisa ser meio termo. Poder ser um terço de termo ou outra fração qualquer. O que não se pode é pensar apenas em si mesmo e nem não pensar em si mesmo.

Na equação do coefieciente angular de uma reta; M=tg do ângulo alfa, o (m) é o mesmo que o ((a) coefiente angular) eu poderia trocar o m pelo a? a= tg alfa?

Não é sempre que o coeficiente angular de uma reta é igual à tangente do ângulo que a reta faz com o eixo das abcissas. Apenas quando os eixos são perpendiculares e quando as escalas dos dois eixos são iguais.

Falando por experiência própria, é cansativo ajudar (ou tentar) ajudar as pessoas uma a uma?

Sim, é. Mas o que é que tem ser?

Alguns pagam com a solidão, desprezo, indiferença ou até mesmo com a própria vida quando resolvem ajudar o próximo, quando resolvem realmente viver por isso, é justo?

Não é justo. Mas não se pode deixar de fazer o bem por se levar prejuízo com isso. Se se levar, esse prejuízo é honroso. É um galardão.

Uma funcão f(x)=x é modular? E por quê é uma funcão ímpar, visto que ela é simetrica em relação à origem y=0?

Não é modular e é ímpar, justamente porque é simétrica em relação à origem. Para ser par, não seria simétrica em relação à origem e sim em relação ao eixo dos y.

É errado pensar em mudar o mundo de uma forma grandiosa? Ou o certo é ajudar aos poucos, um por um..

Não é errado. Só que, geralmente, não se consegue. Então, para se fazer algo a respeito, tem-se que ir aos poucos.

A sua esposa e seus filhos algum dia criticaram seu modo de ser, de ajudar os outros, em algum momento eles se sentiram como se o senhor os tivessem colocado em segundo plano?

Não. Eles também são assim. Especialmente minha mulher. E eu não os coloco em segundo plano, de modo nenhum.

Desde que me entendo por gente sempre quis aprender para ensinar. E sempre fui um professor, mesmo quando ainda era aluno. Estudava para ensinar aos colegas.

Costuma-se chamar de y (mas pode ser qualquer letra ou símbolo) o valor numérico que toma uma função que leve um conjunto numérico a outro de acordo com alguma regra, normalmente definida por meio de uma expressão algébrica que opera sobre um outro valor numérico variável, geralmente chamado de x (mas que pode, também, ser representado por qualquer letra ou símbolo). F(x) é, justamente, a expressão que permite, para cada valor de x, encontrar o correspondente valor de y. Dizer que y = F(x) é dizer que o valor y é obtido ao se operar sobre o valor x de acordo com o que a função F estabelece.

O senhor começou seus estudos pensando em si mesmo ou já começou a aprender o que sabe hoje pensando em passar isso adiante, pensando em usar o que aprenderia para o bem das pessoas a sua volta?

Desde que me entendo por gente sempre quis aprender para ensinar. E sempre fui um professor, mesmo quando ainda era aluno. Estudava para ensinar aos colegas.

O senhor já está mudando o mundo?

Acho que sim, pelo menos um pouco. Como testemunham muitos de meus ex-alunos e seguidores da internet, que afirmam terem mudado sua cosmovisão e seu procedimento em razão do que digo. Do mesmo modo que fui convencido por um internauta, o animadruga, a me tornar vegetariano.

Ajuda é pra quem merece ou pra quem precisa?

Para quem precise.

O que acha dessas regras de Arnold Schwarzenegger para se obter sucesso:Confie em si mesmo, Quebre as regras, NÃO tenha medo de falhar,Não ouça os pessimistas,Trabalhe duro,Retribuir.

Bons conselhos, de fato. Desde que se entenda por sucesso a consecução dos objetivos da pessoa que sejam conformes ao bem do mundo e não a seu próprio, se for contrário ao bem geral.

Para ajudar as pessoas é preciso impor as nossas verdades sob elas?

De modo nenhum. Não se pode impor nada. Tem-se que convencer as pessoas para que elas resolvam, por sua conta, pensar, sentir e agir do modo correto, ajudadas por nós.

Então para mudar o mundo é preciso ter sabedoria, amor , coragem e bravura? São muitas qualidades para se encontrar numa pessoa só, por isso vai demorar para o mundo mudar, pode dar um exemplo de uma pessoa com todas essas caracteristicas?

Claro que não é fácil e é preciso ser uma pessoa excepcional mesmo. E não apenas uma pessoa, mas várias. Exemplo de pessoas assim: Buda, Jesus, Gandhi, Mandela, Churchill, Martin Luther King e outras do tipo. Elas promoveram grandes mudanças no mundo. Mas essas mudanças, de fato, demoram muito para acontecer. Séculos, até.

O senhor é insosso?

Claro que não. Sou uma pessoa, até, bem espirituosa, como podem atestar os que me conhecem pessoalmente. O que não sou é gaiato e caricato e, mesmo, não sou brincalhão e divertido. Mas sou bem alegre, apesar de sério. Isto é, não sou casmurro.

Só o conhecimento liberta o homem?

Não. Uma pessoa pode ser completamente livre e não ter muito conhecimento. O que liberta é o discernimento, a vontade e a decisão. E para decidir corretamente é preciso ter sabedoria, que não significa, necessariamente, sapiência, isto é, conhecimento. Todavia o conhecimento colabora, e muito. Bem como a inteligência.

LinkWithin

Related Posts with Thumbnails